Sunteți pe pagina 1din 79

Basic Income Taxation Fall 2011 | Prof. McCormick Chapter 1: Taxation of Individual Income 1.

. A Brief History of Federal Income Tax a. Ratification of the 16th Amendment in 1913 gave Congress the power to tax: The Congress shall have the power to lay and collect taxes on incomes, from whatever source derived, without apportionment among the several states and without regard to any census or enumeration. b. The point of taxing is for operating funds for the government, allocates the cost of public goods and services among Americans on an ability to pay basis. c. If the Commissioner of the IRS asserts that a deficiency in income tax exists, a taxpayer may: i. Refuse to pay the tax and petition the Tax Court for a redetermination of the deficiency; OR ii. Pay the deficiency, file an administrative claim for refund, and upon denial of the claim, sue for refund in federal district court or the US Court of Federal Claims. 2. Resolution of Tax Issues Through the Judicial Process a. Three courts have original jurisdiction in federal tax cases: the Tax Court, the US District Courts, and the US Court of Federal Claims. i. Tax Courts 1. Referred to as the poor mans court because tax payers can bring action in this court without first paying the asserted deficiency. 2. Cases are heard by a judge, without a jury. The judge then submits the decisions to the Chief Judge for consideration. a. Chief Judge will either allow the decision to stand or refer it to the full court for review. i. Reviewed opinions of the Tax Court are likely to be accorded greater weight. ii. Published opinions of the Tax Court always indicate whether the opinion has been reviewed. iii. Dissenting opinions are published. ii. Federal District Courts 1. Disputed amount must be paid first, then suit follows. 2. May be tried and heard before a jury. 3. Can hear a case in the jurisdiction in which the taxpayer resides, or in the case of a corporation, in the district in which it has its principal place of business. iii. US Court of Federal Claims 1. No jury trial available. 2. Can only hear refund suits, no jurisdiction to hear deficiency cases. 3. Principle office is in DC, but the court may hold hearings in places and times as it sees fit (where appeals of the lower courts are heard typically). b. Appeals i. Appeals from the Tax Court hare heard as a matter of right by the Federal Court of Appeals of the US. 1. In Golsen v. Commissioner, 1970, the Tax Court reversed itself and announced that it would follow a decision of the federal court to which an appeal from a Tax Court decision would be

Basic Income Taxation B | 1

Basic Income Taxation Fall 2011 | Prof. McCormick made, if the federal appeals court decisions were squarely on point. 2. Decisions of the Court of Appeals for the Federal Circuit Court are reviewable by the Supreme Court. 3. Analysis of the Computation of Tax Liability a. Basic questions addressed by an income tax system: i. What items of economic income or gain will be includable in gross income? ii. What costs will be allowable as deductions? iii. When is an amount included in income? When is the taxpayer entitled to claim a deduction for an amount that is deductible? iv. Who is the taxpayer who is going to be taxed on items of income? v. What is the character of the items of income or the deductions? b. Overarching formula: i. Gross Income Deductions = Taxable Income 1. Gross Income (61) except as otherwise provided in this subtitle, gross income means all income from whatever source derived, including (but not limited to) the following items: a. Compensation for services, including fees, commissions, fringe benefits, and similar items; b. Gross income derived from dealings in property; c. Interest; d. Rents; e. Royalties; f. Dividends; g. Alimony and separate maintenance payments; h. Annuities; i. Income from life insurance and endowment Ks; j. Pensions; k. Income from discharge of indebtedness; l. Distributive share of partnership gross income; m. Income in respect of a decedent; and n. Income from an interest in an estate or trust. ii. Gross Income - Above the Line Deductions (itemized deduction) = Adjusted Gross Income (62) a. Adjusted Gross Income means, gross income minus the following deductions: i. Trade and business deductions; ii. Certain trade and business deductions of employees: 1. Reimbursed expenses of employees; 2. Certain expenses of performing artists; 3. Certain expenses of officials; 4. Certain expenses of elementary and secondary school teachers; iii. Losses from sale or exchange of property; iv. Deductions attributable to rents and royalties; v. Certain deductions of life tenants and income beneficiaries of property; vi. Pension, profit-sharing, and annuity plans of self-employed individuals;

Basic Income Taxation B | 2

Basic Income Taxation Fall 2011 | Prof. McCormick vii. Retirement savings; viii. Alimony; ix. Moving expenses; x. Interest on education loans; xi. Higher education expenses; xii. Health savings accounts; xiii. Costs involving discrimination suits; iii. Below-the-Line Deductions (Standard Deduction) a. Set by Congress each year and its a fixed amount. The affect is if a taxpayer does not have many deductions, then they do not need to keep records and will take the standard deduction. iv. Either way, if taking a standard or itemized deduction, ALWAYS subtract personal exemptions (like dependents). = TAXABLE INCOME (63) Deductions i. Must find a specific provision in the code for an itemized deduction. ii. If a taxpayers below-the-line deductions exceed the standard deduction, the taxpayer should be advised to itemize. 1. (a) For Itemized taxes: Taxable income means gross income minus itemized deductions and personal exemptions. 2. (b) For Standard deduction taxes: Taxable income means adjusted gross income, minus the standard deduction and the deduction for personal exemptions. Credits i. Credits are better than deductions time value of money ii. If on the border of brackets, should you take a deduction or credit? 1. CREDIT!!!! iii. $1000 deduction means more to a higher tax bracket payer than a lower. iv. Look in Sections 21-53 v. 31 is the most common. It is the credit for withholding taxes paid through the year by employers on behalf of the employees. Marginal Tax Rate: i. Rate on last dollar of income earned 1. i.e. $89,500 = 31% (see the table!) Effective Tax Rate: i. Average of all tax rates, and determines what percent each dollar of income is taxed. 1. i.e. $89,500 / 14,400 = 25% 2. Tax Liability / Taxable Income = effective tax rate

c. d.

e.

f. g.

Chapter 1 Problems: 1. Tom & Caroline Taxpayer use the cash method of accounting and report their income on a calendar year basis. Tom is a business consultant who owns and operates his own business. Caroline is a university professor. What is their tax liability?

Basic Income Taxation B | 3

Basic Income Taxation Fall 2011 | Prof. McCormick Gross Income: Carolines University Toms Fees Service Exchange Interest Income Dividends Stock Sale Gain GI $125,000 150,000 10,000 19,000 1,000 15,000 $320,000

Adjusted Gross Income: determine which deduction they should take! Standard Deduction Above-the-Line Deductions: Bus. Expenses Wages Bus. Expenses Various Building Depreciation TOTAL $60,000 20,000 10,000 $90,000 $11,400

Below-the-Line Deductions: (in order to take must exceed 2% of GI 67(a)) Management Fee House Interest State Income Tax Real Property Taxes Donations $1,000 18,000 14,000 4,400 9,000 $46,400 doesnt exceed 2% 67(b)(2) 67(b)(2) 67(b)(4) $45,400 Total Itemized AGI = GI (above-the-line) $320,000 90,000 = $230,000 AGI (itemized & Personal exemptions) USE Table in the back (Page 1985), use 1(f), if no years are given use 1(i) page 22. Chapter 2: Gross Income Concepts & Limitations 1. The answer to defining income is found in an examination of the interplay of congressional purpose, administrative goals, accounting concepts, and public policy as defined by the Congress, the courts and the Treasury. 2. Defining Income a. Gross income is defined in 61 b. Glenshaw Glass: Congress in defining gross income intended to exert the full measure of its taxing power. i. Income is any accession to wealth clearly realized and over which the taxpayer has dominion. (aka: if it makes you wealthier it is income)

Basic Income Taxation B | 4

Basic Income Taxation Fall 2011 | Prof. McCormick c. Eisner v. Macomber: the Court described income as the gain derived from capital, from labor, or from both combined, provided it be understood to include profit gained through a sale or conversion of capital assets. d. Hawkins v. Commissioner: indicated that Eisners definition of income is incomplete. e. U.S. v. Kirby Lumber the discharge of a corporate debt for an amount less than the face of the debt resulted in as income to the debtor. 3. Income Realized in Any Form a. Reg 1.61-1(a): explicitly provides that gross income may be realized in any form, whether money, property, or services. b. Reg 1.61-2(d)(1): if services are paid for in property, the FMV of the property is the measure of compensation; if paid for in the form of services, the value of the services received is the amount of compensation. i. FMV is defined as the price a willing buyer would pay a willing seller, with neither under a compulsion to buy or sell, and both having reasonable knowledge of relevant facts. c. A tax system cannot depend on the taxpayers receipt of cash as the sine qua non of income. Measurement may be much less certain when valuation of goods or services is necessary, but the difficulty of measurement should not obscure the fact of payment. i. NOTE: A taxpayer who accumulates frequent flier miles as a result of business travel paid for by his or her employer is not required to report any gross income as a result of the receipt of the frequent flier miles or of her use of those miles for her personal travel. 4. Realization, Imputed Income, and Bargain Purchases a. The way you label these are important, very different tax implications!!! b. Gross Income does not encompass all accessions to wealth. i. Stock dividends are not an increase in income. 1. Eisner v. Macomber a. Not realized yet. Issue of timing. c. Realization the application in value of some assets will not be charged with income until suck realization occurs. Realization events are not limited to cash sales. i. Conversion of property into cash or the exchange of property for other property differing materially either in kind or in extent also counts as realization. 1. Two Reasons for Realization Method: a. Measuring the appreciation in all of the property every taxpayer every year would present enormous administrative problems for taxpayers and the IRS. b. It is often claimed that it would be fundamentally unfair to treat unrealized gains as income because taxpayers might well lack the case to pay resulting taxes and might thus be forced to sell assets to pay the tax. ii. An exchange of property gives rise to a realization event so long as the exchanged properties are materially different that is, so long as they embody legally distinct entitlements. d. Barter Transactions i. Revenue Ruling 79-24 (page 41) 1. Legal services traded for greenhouse a. Both are taxable for the FV of the services.

Basic Income Taxation B | 5

Basic Income Taxation Fall 2011 | Prof. McCormick b. Brother has to report the price of the legal services. c. Marcella has to report the price of the greenhouse. 2. Could Marcella and the brother argue gifts? a. How do you decide barter transaction or gift? i. Have to get into minds and look at the facts and circumstances provided. e. Imputed Income i. NOT TAXED! ii. Self-help income, from performing services for oneself, ones family, or others. Usually associated with self-employment or employment activities. iii. Two categories of imputed income: 1. Imputed income from services 2. Imputed income from property iv. Case Law: 1. Morris v. Commissioner the Board of Tax Appeals stated the value of farm products consumed by the owner was not income. 2. Commissioner v. Daehler the taxpayer, a real estate sales man, purchased property listed with a broker who was not his employer. Tax Court held that the taxpayers commission was income to him, not a reduction in purhase price.s 3. Commissioner v. Minzner held an insurance agent could be taxed on the commissions he received on life insurance he purchased on his own life. f. Bargain Purchases i. Bargain purchases do not constitute income, however, if property is transferred as compensation for services in an amount less than FMV, the difference between the FMV and the amount paid is gross income. Reg 1.61-2(d)(2). ii. The FMV of the asset is greater than the price you paid for it. 1. Pellar v. Commissioner confirmed that bargain purchases generally do not constitute gross income. a. Under Reg. 1.61-2(d)(2)(i), the property is transferred as compensation for services in an amount less than its FMV, the difference between the FMV and the amount paid is gross income. 2. Commissioner v. Glenshaw Glass Co money received as punitive damages must be included as gross income. a. Income is an accession to wealth clearly realized where taxpayer has complete dominionbecause the money is in complete dominion of the company and accession of wealth is clearly found. 3. Cesarini v. US unless expressly excluded by law, gross income includes all income from whatever source derived. a. The Cesarinis after finding money in a piano and declaring it on their taxes, filed an amended tax return that removed the funds in question from gross income. i. Found the money in treasure trove. The source deriving income must be explicitly exempted to not be included in gross income.

Basic Income Taxation B | 6

Basic Income Taxation Fall 2011 | Prof. McCormick 4. Old Colony Trust v. Commissioner The Commissioner sought to tax as additional income to the employee, the amount of his federal income taxes that were paid on his behalf by his employer. a. The payment by an employer of the income taxes assessed against his employee constitutes additional taxable income to the employee. 5. Revenue Ruling 79-24, 1979 a. Compensation for services 61(a) and 1.61-2 b. 1.61-2(d)(1) of the regulations provides that if services are paid for other than in money, the FMV of the property or services taken in payment must be included in income.x 6. McCann v. US McCanns received an all-expenses paid trip as a reward for Mrs. McCanns job performance, but they did not include the value of the trip in their gross income calculation on their joint income tax returns. a. When services are paid for in a form other than money, the FMV of the thing received must be included in gross income. i. The Convenience of the Employer Doctrine not going for employers convenience, but sometimes they have employees go on trip because its good for business. 1. Argue good for business? a. Good for morale, able to meet all other lawyers at firm. b. Have to only pay for training once. 2. Improving morale counts as income! 3. Therefore, when a company pays travel
expenses, a taxpayer must include such compensation in gross income when the excursion is viewed as a reward for outstanding employee success within the company.

7. Pellar v. Commissioner Pellars paid $55,000 for a house with a FMV of $70,000. They paid no taxes on the $15,000 difference they realized. a. Court held that the purchase of property for less than its FMV does not, of itself, give rise to the realization of taxable income. i. BUT we have to scrutinize the transaction and investigate the intent of the parties. ii. Section 61 includes fringe benefits as income, unless there is a provision that excludes it. 1. 119, 132 exclude fringe benefits from income. 2. 132 excludes employee discount iii. Be careful not to misapply the realization requirement in connection with a compensatory bargain purchase that constitutes gross income.

Basic Income Taxation B | 7

Basic Income Taxation Fall 2011 | Prof. McCormick 1. Example: Assume an employer transferred to an employee, as compensation, stock worth $500 in exchange to payment of $100. Reg 1.62-2(d)(2)(i) provides that employee has a gross income of $400. a. It would be incorrect to argue that the employee had not income because the $400 gain had not been realized and would not be realized until the employee disposed of the stock. The $400 difference simply does not constitute appreciation in the stocks value while in the hands of the employee. The property was given to the employee as compensation and it is the FMV of that property that must be used in order to measure the compensatory element in the transaction.

Chapter 2 Problems: 1. Marcella a law grad is an associate with a firm. Which needs to be reported as gross income? (a) Salary of $75,000 per year. = YES Take home pay of $50K, firm withheld $15K for taxes (tax credit), and $5.5K in Social Security taxes and $4.5K (will offset) for state income taxes (tax credit). Take home pay is taxable. Withholdings = NO, they are already taxed.

Gross Income is 75k-----Comes from Old Colony Trust Case. (if an employer pays an obligation for you, it is income) (the form of payment makes no difference) (b) (c) A year-end bonus = yes. See 61(a)(1) and Treas. Reg. 1.61-2(a). Antique oak desk worth $500, selling for $50 from her law firm.

Answer = Maybe, is it compensation for services? If it is just a bargain purchase, then no, not income. (Peller case-----if you purchase something at a bargain and there is no employer-employee relationship, dont have to report income) (however, in this problem there is an employee-employer relationship, therefore have to look at other facts of whether or not there is compensation.) (d) Firm retreat mandatory for all to attend in Aspen. Answer = maybe, look to the convenience of employer doctrine. Mcann----it was mandatory for her to go; however, in this case it was not required, therefore there is an argument for her not to include it in income. (e) Small greenhouse worth $2,500 constructed on Marcellas property by her brother Bill.

Basic Income Taxation B | 8

Basic Income Taxation Fall 2011 | Prof. McCormick ANSWER: See Pellar case or if it is really a gift, then not income. Also See Rev. Ruling 79-24, which states that in a barter transaction, FMV is the income. (report the fair market value of what the person receives) Barter transaction rule * does the barter transaction make sense: (f) Imputed income rule----imputed income is not consider income for tax purposes (both self-provided income & self-provided assets). Because she provided a service for herself. (Policy: administratively difficult.) Problem 2: He didnt realize the gain yet up until year 5..In year 5 he realized the gain of $2,000 gain

Chapter 3: The Effect of an Obligation to Repay 1. Loans a. Loans do not count as gross income. 1. If a 3rd party pays off the loan for you, then you have income at that point. ii. A loan does not represent an accession to wealth or increase the taxpayers new worth because the loan proceeds are accompanied by an equal and offsetting liability: the borrower has an obligation to repay the loan, and it is the repayment obligation that negates treatment of a loan as income. iii. The determination of whether a transfer of funds constitutes a loan is a question of factin order for a transfer of funds to constitute a loan, at the time the funds are transferred there must be an unconditional obligation on the part of the transferee to repay, and an unconditional intention on the part of the transferor to secure payment, of such frauds. 1. Whether a transfer for funds constitutes a loan may be inferred from objective characteristics surrounding the transfer, including the presence or absence of a debt instrument, collateral securing the purported loan, interest accruing on the purported loan, repayments of the transferred funds, and any attributes indicative of an enforceable obligation on the part of transferee to repay the funds transferred. b. Repayment of a loan does not reduce gross income. i. However, payment of ones liabilities by another may give rise to gross income.discharge of indebtedness. 2. Claim of Right a. The Claim of Right doctrine: if a taxpayer receives earnings under a claim of right (money you get and money you are entitled to receive) and without restriction as to its disposition, he has received income, which he is required to return (that is, to report on his tax return), even though it may still be claimed that he is not entitled to retain the money, and even though he may still be adjudged liable to restore its equivalent. See, e.g., North American Consolidated v. Burnet.

Basic Income Taxation B | 9

Basic Income Taxation Fall 2011 | Prof. McCormick i. If properly claimed, and the taxpayer is subsequently required to refund the money, the taxpayer is entitled to a deduction (or a reduction under 1341). Also a tax credit option!!! ii. Money received under a claim of right, without restriction as to disposition, is income; the contingent repayment obligation does not allow the receipt to be treated as a loan. iii. If the taxpayer sets aside the amounts received, retains control over them, and elects not to use them pending resolution of the dispute, it still counts as income. See, e.g., Commissioner v. Alamitos Land Co., and Revenue Ruling 55-137, 1195-1 C.B. 215. 1. If told to set aside possible excess commissions collected, the amounts deposited by court order are not income. 2. Funds over which the taxpayer acts as a conduit (a channel for conveying something) are not received under a claim of right. 3. Illegal Income a. Gains from an illegal business may be taxed. b. Embezzlement has an obligation like a loan to the organization it stole from; it is treated as income though. i. Repayment of illegal income entitles the taxpayer to a deduction. ii. Crimer (spelling?) case look to the intent of the borrower and decide what label to call it in regards to a loan (repayment intent) or an embezzlement (no intent to repay) 1. If this is reported and repaid, then the taxpayer takes a deduction for paying illegal income. 2. Generally the majority view---look to the intent of the party at time they took the money. Or if the intent of the parties change during the process and they no longer want to the pay the loan then it is an embellzement and will be considered income when the intent change. 4. Deposits a. The regulations explicitly provide that rent paid in advance generally constitutes gross income in the year it is received regardless of the period covered or the taxpayers method of accounting. Reg. 1.61-8(b). i. An advance payment of rent is still income; if truly a deposit (ex. a security deposit), not income. b. Generally, a good faith showing typically not income. c. In Indianapolis Power & Light, the Supreme Court held that control over the conditions of refund is the determinative factor in deciding whether a deposit will be taxable as an advance or nontaxable as a deposit. i. If the payor controls the conditions under which the money will be repaid or refunded, generally, the payment is not income to the recipient. ii. On the other hand, if the recipient of the payment controls the conditions under which the payment will be repaid or refunded, the recipient has some guarantee that it will be allowed to keep the money, and hence, the recipient enjoys complete dominion over the payment. d. The mere fact of refundability is not in itself sufficient for identifying nontaxable deposits, refundability must be within the buyers controls. e. The complete dominion or control standard of Indianapolis Power & Light was applied in the rental context in Highland Farms. f. An advance payment is one, which protects against the risk that the buyer will back out before the taxpayer has a chance to perform, the pre-need contracts and

Basic Income Taxation B | 10

Basic Income Taxation Fall 2011 | Prof. McCormick payments fail to serve that function because of the open-ended nature of the cancellation right. i. Count as income when received, like a cash-basis taxpayer. g. Case Law: i. North American Oil Consolidated v. Burnet funds impounded by a receiver who is in control of only a portion of a corporations property may be taxed to the corporation only when the corporation has an unqualified right to received them. 1. If a taxpayer receives earnings under a claim of right and without restriction as to its disposition, he has received income that he is required to return even though it may still be claimed that he is not entitled to retain the money, and even though he may still be adjudged liable to restore its equivalent. ii. James v. United States------When a taxpayer acquires earnings, lawfully or unlawfully, w/o the consensual recognition, express or implied, of an obligation to repay and without restriction as to their disposition, he has received income. iii. Commissioner v. Indianapolis Power & Light Company, customer deposits are not income where the taxpayer receiving the deposits has an express or implied obligation to repay the deposits and lacks complete dominion over their disposition. Chapter 3 Problems: 1.) Kevin is a shoe department manager and he is paid monthly. His monthly paycheck includes his regular salary and a commission equal to 3% of the shoe departments gross sales in excess of $250K during the previous month. Kevin has earned commissions averaging $1,500 per month. December paycheck included a commission of $3K based on the November sales. In January, Kevin was advised that a mistake had been made in the computation for the November sales commission and the amount of the December check should have been $1K less. To rectify the situation, the payroll department reduced Kevins compensation for January by $1K. (a) How should Kevin report the $3K received in one year, and the repayment of the $1K of that commission in the next year? (Option 1) (Option 2) Two ways to possibly report: year 1 = $3K year 1 = $2K year 2 = ($1K) (take deduction) year 2= $0

Correct Answer: Option 1. Must report actual income in Year 1 under the claim of right doctrine from North American Oil Consolidated, ruling that as a claim of right, the income must be reported as $3K despite the possibility that the income may need to be disgorged as a deduction in the following year. Also see, 1341(a)(4) If you receive money in a year, believing your entitled to it, you must report the income in the year it was received.

Basic Income Taxation B | 11

Basic Income Taxation Fall 2011 | Prof. McCormick Redress? Take deduction in year 2, or a credit for extra taxes from year 1. (b) Kevin recognized that a mistake had been made in computing his commission. Kevin did not report the mistake to the payroll department, but Kevin viewed the matter as an interest-free loan because he knew the store would correct the mistake eventually. In January, Kevin was required to repay the $1K. In April, when Kevin filed taxes, he reported $2K of the $3K commission. Is this proper? A loan is not taxable. He cannot use claim of right doctrine because he had intent and knowledge to use and knew it wasnt his. He had no entitlement to the money. IRS would treat as income. Rule is to look at the intent of the borrower. Here, his filing was improper. He must report the income in Year 1. 2.) Mark owns his own garbage business. Mark sends bills to his customers on the first of every month, and if he has not received payment within 20 days of sending a bill, he sends a second billing statement to the customer. This means that sometimes customers pay twice for the same bill. If that happens, Mark shows a credit balance on the account and applies the credit balance toward the future bill. If a customer who has overpaid requests a refund of the overpayment, Mark sends it to them. Typically, customers dont request the refund. At the end of December of the current year, Mark holds a total of $5K in customer overpayments which he has credited to customer accounts. He did not report in Gross Income for the year, but plans to report the $5K on his return for the following year when realization actually occurs. Citing the claim of right doctrine, the IRS challenges Marks failure to report the $5K in gross income on this years return. Assess the position taken by the IRS. ANSWER: This is a deposit versus advance payment question, so the determinative issue is the timing of the payment and the control associated with that payment. IRS wants to treat as income in Y1, thus it is an advance payment and is taxable. Taxpayer would want to consider it a deposit. Indianapolis Power & Light control standard. Who has legal control over the money to be refunded? Here, the customer has the ultimate authority and is able to request the money, thus, it is not an advance payment, but merely a deposit.

Basic Income Taxation B | 12

Basic Income Taxation Fall 2011 | Prof. McCormick 3.) Kevin owns homes and rents them to University students. Requires a security deposit worth one months rent. Rental agreement used by Kevin allows that the security deposit will be applied (a) to compensate Keving for any property damage, or (b) to cover any unpaid rent. Kevin does not maintain a separate account or pays interest on the security deposits. Kevin usually will apply a security deposit to the last months rent or for damages. He rarely returns a security deposit. How should you advise Kevin to treat the security deposits for tax purposes? If treated as: (1) Advance Payment = income when received. (2) Deposit = not income, belongs to the renter I.E. Sept Y1 Aug Y2 Security Deposit Taxed in Y2 **TAXPAYER WANTS**

Advanced Payment Taxed in Y1 **IRS PREFERS** Analysis:

Who controls the conditions of the refund under the contract? Indianapolis Power & Light if payor = deposit; if recipient = A.Paym Because the tenants have the choice under the contract, they have control and thus, it is a deposit and not an advance payment.

4.)

Karen offers a course designed to prep individuals to take a real estate exam. Karen charges $1.5K tuition one month before the course commences. Karen agrees to refund the full $1.5K to any student who successfully completes her course but fails the state examination. Based on previous experience, Karen assumes less than 5% of students taking her course will fail the state examination. Karen deposits student tuition in her personal checking account. When must Karen report as gross income the tuition she receives from her students the year of prepayment or the following year when she knows the exam results? ANSWER: Claim of Right Analysis: receive money & entitled to it even if theres a possibility of return Must report in income. Advance Payment v. Deposit: - Check the contract, who controls by its terms? - agreement with students is that she gets the $ unless they fail the test. Students are paying a deposit because tuition can be

Basic Income Taxation B | 13

Basic Income Taxation Fall 2011 | Prof. McCormick returned if/when they fail exam. Counted as income in Y2 with deductions. Schnitt v. Commissioner law professor offering bar review classCourt held that law professor needed to include the income in Y1 even though he had to repay students who failed bar exam. Chapter 4: Gains Derived From Dealings in Property 1. According to 61(a)(3), gross income includes gains derived from dealings in property. 2. Reg. 1.61(6)(a) provides gain is the excess of the amount realized over the unrecovered cost or other basis for the property sold or exchanged. In addition, the specific rules for computing the amount of gain or loss are contained in 1001 and the regulations there. a. 1001(b) provides that the amount realized on the sale or other disposition of property equals the money received plus the FMV of any other property received. b. 1012 basis = cost except as otherwise provided c. 1016 adjusted basis = cost + cost of improvements i. Referred to as the recovery of capital or return on capital concept: basis and adjusted basis implement the recovery of capital concept by providing a measure of the capital (or investment) the taxpayer is entitled to recover tax-free. 1. Also applicable to the borrowing concept. d. 61(a)(7) provides that dividends constitutes gross income. A dividend is viewed as earning profit on ones investment much the way rent represents earning on ones property or interest represents earnings. 3. Tax-Cost Basis a. Reg 1.61-2(d)(2)(i) essentially, whatever the item cost is the basis the taxpayer takes in the acquisition of the property. 4. Impact of Liabilities a. Impact on Basis: i. Commissioner v. Tufts, the Court held that because of the obligation to repay a loan, the taxpayer is entitled to include the amount of the loan in computing his basis in the property. The loan, under 1012, is part of the taxpayers cost-basis of the property. 1. By reflecting in basis the debt incurred by a taxpayer in acquiring property, the tax system gives credit to a taxpayer for an investment the taxpayer has yet to make. b. Impact on Amount Realized: i. Amount realized is defined in 1001(b). 1. Amount Realized = Basis 2. AR = Cash Received + FMV of any property rcd + amount of debt new buyer takes over ii. Determination of the amount realized by a seller is likewise not straightforward when the property sold is encumbered by liabilities for which the purchaser directly or indirectly becomes responsible. iii. Reg 1.1001-2(a)(1): a necessary corollary to the inclusion of liabilities in basis is the inclusion in amount realized of those liabilities of the taxpayer assumed by the purchaser. iv. UNDERSTAND TWO GENERAL RULES: 1. Recourse liabilities incurred by a taxpayer in the acquisition of property are included in the taxpayers basis in that property; and

Basic Income Taxation B | 14

Basic Income Taxation Fall 2011 | Prof. McCormick 2. Recourse liabilities of a seller, assumed by a purchaser, are included in the sellers amount realized. 5. Basis of Property Acquired in Taxable Exchange: a. Philadelphia Park Amusement Co. v. US the court held that the cost basis of the property received in a taxable exchange is the FMV of the property received in the exchange. b. However, in case of properties with different value being exchanged, the cost basis of the property received in a taxable exchange is the FMV of the property received in the exchange. 6. Rules Worth Mentioning: a. Amount of recourse debt borrowed to finance property is included in the basis and included in the cost. b. Debt on property that is assumed by purchaser is included in the sellers amount realized. c. Taxable exchange of property (a trade), the basis of the property received in exchange is the FMV of the property received on the day it was exchanged.

Chapter 4 Problems: 1.) S purchased a 5-acre plot of land for $100K. Sold the entire tract yesterday for $300K. How much income should S report? Gain = Amount Realized Adjusted Basis x = $300K - $100K x = $200K What if S subdivides the tract into 5, 1-acre parcels and sells each parcel for $75K? Reg 1.61-(6)(a): allocate basis in a reasonable way given the facts and surrounding circumstances. $75K x 5 = $ 375K Amount Realized - 100K Adjusted Basis $275K Gain

2.)

Maggie purchased a summer home for $500K. She used $100K of cash, and borrowed $400K. (a) Basis in the summer home? It cost $500K, so basis is the same (b) She paid off part of the loan, owes now only $300K. What is Maggies basis? Still $500K. (c) Reduced the mortgage balance to $250K. She refinanced the home, Maggie borrowed $250K, thus her total mortgage is $500K. Maggie used $75K to

Basic Income Taxation B | 15

Basic Income Taxation Fall 2011 | Prof. McCormick remodel the home; $125K to purchase land to hold for investment, $50K for a family vacation. What is Maggies adjusted basis in the summer home? AB = cost of the home = $500K + $75K for remodeling. Basis in the land she newly purchased? $100K. (d) Sold the home. Purchaser paid $250K in cash and assumed the balance of $450K, which encumbered the home. How much gain will Maggie realize on the sale? Gain = x = x = x = x = 3.) Amount Realized Adjusted Basis (cash rcd + FMV of property + debt) AB ($250K + 0 + $450K) - $575K $700K - $575K $125K of gain

Clare owes Liz $6K in legal fees. Liz purchases one of Clares paintings for $6K. Clare then uses the $6K to pay Liz back. Five years later, Liz sells the painting for $10K. What tax consequences? Gain = AR AB x = $10K - $6K x = $4K Alternatively, assume Liz, instead of paying Clare $6K, agrees to accept the painting as payment in full for her legal services. Liz then sells the painting, what are the tax consequences? Same thing. $4K in gain. This is a barter transaction. Both parties have income; the value of the property received becomes the basis. See Philadelphia Park.

4.)

Katie transferred to Pat an undeveloped 5-acre tract of land in Arizona in exchange for an acre of undeveloped lakefront property in Nevada. Katie had a $150K adjusted basis in the tract of land that was worth $450K at the time of the exchange. Patrick had a $50K adjusted basis in his Nevada lake property, which also had a FMV of exactly $450K. (a) What tax consequences to Katie and Pat on the exchange? Gain and Basis? Basis: Gain: Katies is $150K; Pats is $50K. AR AB Katies = $450K - $150K = $300K Pats = $450K - $50K = $400K New basis in each property is $450K.

Basic Income Taxation B | 16

Basic Income Taxation Fall 2011 | Prof. McCormick (b) Now, Katies land has a FMV of $500K and Pat transferred to Katie his Nevada lake property plus $50K in cash? Gain: Katies = $500K - $150K = $350 K Pats = $500K - $100K ($50K basis + cash paid, like making a fair trade) = $400K New Basis: Pats = $500K, the FMV of the property when he received it. (Philadelpha park case) Katies = $450K (FMV of the property when she received it). This does not take into account the cash that Pat paid her because it does not actually add to the basis of the home. AR= 50k + 450k = 500k (see Gain above) AB = 150k (c) Pat assumed a $50K mortgage instead of paying Katie $50K in cash. Pat sold $550K, but paid $10K on the mortgage of his home that he received from Katie. The purchaser paid Pat $510,000 in cash and assumed the $40K balance on the mortgage that encumbered the property. Tax consequences to Pat on the sale of the land? Pats Gain: $450K (FMV of AZ land minus mortgage) - $50K = $400K

AR (500k -50k = 450k) AB (50k) = Gain Pats Basis: FMV of land is $450K (considering Philadelphia Park). However, general case law also includes the debt in the basis, and thus the total basis is $500K. (Recourse debt rule-if you take on recourse debt while taking on property you are allowed to tact that debt in your basis) Katies AR = $500K, the FMV of the AZ lake property, which is $450K plus the amount of debt that Pat takes over from her. AB = $150K (her original basis in the land) Gain = $350K. (income for the year) Katies New basis: $450K, which is the FMV of the NV property. Tax Consequences: AR = $510K + $40K of remaining mortgage = $550K AB = $500K (this includes the $450K FMV of land + the $50K loan) Gain = $50 K Chapter 5: Gifts, Bequests, Devises & Inheritances 1. Gifts are not income. Determined on a case-by-case basis, focus on the donors intent. 2. What is excluded under 102? a. Excludes gifts, as well as property, acquired from a decedent through bequest, devise, or inheritance.

Basic Income Taxation B | 17

Basic Income Taxation Fall 2011 | Prof. McCormick i. Property we receive as a result of the generosity of a person either during his lifetime (inter vivos gift) or at his death is excluded from income. b. Threshold Question: What is the donors intent? i. Commissioner v. Duberstein a gift in the statutory senseproceeds from a detached and disinterested generosityout of affection, respect, admiration, charity or like impulsesand in this regard, the most critical considerationis the transferors intention. 1. In determining whether transferred property constitutes a gift, the trier of fact must analyze all relevant factors, rather than relying solely on the transferors or transferees subjective intent. ii. Goodwin v. US Reverend and wife were paid to stay at the church, kind of like an incentive. The court held that regular, sizable payments made by persons to whom the taxpayer provides services may be regarded as a form of compensation and therefore as taxable income. c. 102(a) states that amounts transferred by an employer to, or for the benefit of, an employee are not excludable. i. 247(b) disallows a deduction for gifts to individuals in excess of $25. 1. A business making gifts is faced with a choice: on the one hand, it can transfer property to an individual and characterize the transfer as compensation and presumably entitle itself to a business deduction. d. 102(c)(1) negates gift status for transfers from employers to employees. i. Such transfers as a result do not implicate 274(b), which applies only to items excludable as gifts under 274. ii. Exclusion under Employer Achievement Award like a gold watch for 25 years of employment, not taxable! 3. The Nature of a Bequest or Inheritance a. With respect to cash or other property received as the result of anothers death (bequests, devises, and inheritance) a threshold question similar to that in the gift context is raised: Is the cash or property received a bequest, devise, or inheritance, or is it compensation or some other form of taxable income? b. Statutory Limitations on the Exclusion 102(b) i. The income from property excluded as a gift, bequest, devise, or inheritance is not excluded. 1. Thus, if X gives Y a share of stock, the value of the stock is excluded from Ys income, but the dividends which IBM distributes to Y are not. ii. Denies an exclusion to gifts, whether made during life or at death, or of income from property. 4. Basis of Property Received by Gift a. Gifts of Appreciated Property i. Basis Rule = 1015(a) provides that the transferees basis of the property received by gift is the same as the transferors. The effect of the transferred basis rule is to shift the tax burden associated with the appreciated value of the stock to the transferee. 1. Substituted Basis or Transferred Basis a. The transferred basis rule as applied to gifts assures that the appreciation of a gift remains subject to taxation. i. When sold, the appreciation is taxable. See Taft v. Bowers, which confirms that the appreciation

Basic Income Taxation B | 18

Basic Income Taxation Fall 2011 | Prof. McCormick inherent in gifts may be ultimately taxed to the donee. b. Gifts of Property Basis in Exchange of Fair Market Value i. The transferred basis rule (1015(a)) and the rule of Taft v. Bowers enables a donor to shift potential gain to a donee. ii. While permitting the use of gifts to shift income, shifting of losses is prohibited. 1015(a). 5. Basis of Property Received by Bequest or Inheritance a. 1014 governs the determination of basis of property received by bequest, devise, or inheritance. i. Stepped-up (or Stepped-down) Basis: Provision increases or decreases the basis of property acquired from the decedent to the FMV of the property at the time of the decedents death. b. Provides that property acquired from a decedent generally takes a basis equal to the FMV of the property at t/he date of the decedents death. i. Appreciated property is a tax amnesty for the gain inherent in the property at the time of a persons death. The devisee or heir receiving the stepped-up basis can sell the property for its value as of the decedents death and not realize any gain. Only the appreciation occurring after the decedents death will be subject to taxation. 6. Part-Gift, Part-Sale----compare with part gift part sale.. a. The sale of property for less than FMV involves a sale in part and a gift in part. The sale of property for less than FMV is common between family members and friends. b. Reg 1.1001-1(e) states that the seller-donor has gain to the extent that the amount realized exceeds the adjusted basis of the property. No loss is recognized on such a transaction. c. Reg 1.1015-4 provides that the donees basis will be the greater of the amount the donee paid for the property or the adjusted basis of the donor. i. A special rule consistent with 1015(a) limits the donees basis for purposes of computing loss, to the FMV of the property at the time of the transfer to the donee. d. So, if it is a part-gift/part-sale, the donor has to recognize gain if it is greater than their basis. However, if there is a loss, no deduction. e. For the receiver of the deal, we need to know their basis in the property: i. The better of either the 1. Gift basis (1015) 2. Or the Cost Basis (1012)----Cost basis is the amount you paid. a. Whichever is higher is the value used.

Chapter 5 Problems: 1.) Carol is a legal assistant employed by Lucille, a lawyer. At the firms annual holiday party, Carol received substantial cash presents from Lucille, from the associate lawyer, and from one of Lucilles clients. The other legal assistant, worked there less time wise, did not receive as much. Are the cash presents includable? Under 102(c), the cash payment is income, especially because its from an employer. Also, under Commissioner v. Duberstein, the employer was seeking to compensate the employee for their services, and therefore, it is taxable and not

Basic Income Taxation B | 19

Basic Income Taxation Fall 2011 | Prof. McCormick a gift. Other Clients gifts = probably not associated with the employer, thus can be treated like a gift. 2.) Maria lives in a rented apartment in NYC near her mom. Her mom lives alone in a Park Avenue apartment she owns. Marias sisters are married with children and live on the West Coast. Marias Mom becomes frailer and asks Maria to move in with her. Maria agreed, moved in with her mom and took care of her for six years until her mother died. Moms will devised the Park Avenue apartment to Maria, noting the devise was made in gratitude for Marias care. The balance of Moms estate was made to the three children and divided equally. At the time of death, the Park Avenue apartment had a FMV of $2.5 million. What are the tax consequences that Maria will have as a result of the devise to her of the apartment? If compensation, then taxable; if a bequest, not income. Look to the situation surrounding it, is there a reasons that mom may have left it to Maria on purpose (fact that she is single and is the only daughter living on the east coast). What basis will Maria have in the apartment? $2.5M, the value of the property on the date of receipt. 3.) Dan purchased for investment purposes land for $100K. Dan deeded the land to his son, Will, this year as a gift. The land had a FMV of $250K, and an adjusted basis of $100K. (a) Will Dan or Will recognize income on the transfer? Making an intervivos gift is not a realization event, so no income to Dan. Will has no income because it was a gift. What basis does Will take in the land? Will takes carryover basis and upon sale will have to report gain. Basis is $100K. (1015) (b) What if Dan is also Wills employer? Implicates 102(c). Proposed 1.102-1(f)(2) provides that where employee can show that the familial relationship was the reason that the transfer occurred, not the employment relationship, then 102(a) may apply. (c) What if, Dan sold land to Will for $50K? Dan would lose $50K and CANNOT deduct the loss. Part gift-part sale (transaction) Wills basis is $100K. What about for $200K? (still a part gift-part sale transaction) Wills basis would be stepped-up to $200K, the cost-basis. Dan would report a gain of $100K. sold for 200k and purchased for a 100k, therefore gain of 100k.

Basic Income Taxation B | 20

Basic Income Taxation Fall 2011 | Prof. McCormick (d) What if Dans land were encumbered by a mortgage in the amount of $125K and Will agreed to assume the mortgage?

This is a part-gift/part-sale, treat as if you borrowed. Dans Gain = (AR) Cause hes better off without the mortgage $125K $100K (Basis) = $25K gain (1.001-1). Must recognize as gross income. Wills basis -----gift basis (1015---carry-over basis) (100k) or cost basis (125k). Must take larger, therefore Wills basis is 125k.

(e) (exception to 1015, stated in 1015)90k basis for determining loss, 100k basis for determine gain. For these purposes the convention there is no loss nor gain Assume Dans land was only worth $90K when he made the gift of the land to Will. What tax consequences to Will when he sells the land for $90K? No income, Will takes the basis of donor, unless basis is > FMV at time of gift, basis is FMV I.E. Property Depreciation uses FMV for losses Property Appreciation uses Carry-over basis for gains Basis is $90K What if he sells for $80K? Gain = $80K - $90K = ($10K) loss, cannot deduct loss though! What if he sells for $95K? Treat as neither gain or loss: Gain basis = $100K; loss basis = $90K Calculate loss by FMV; gain by Carryover basisby convention there is not gain or loss. RULE: When selling between two people and sale price falls in the middle of the gain basis and the loss basis, treat as if there is no gain or loss. What if he sells for $110K? Gain = $110K - $100K = $10K Gain (f) Dan devised the land to Will and the land had a FMV of $250K at the time of Dans death. What tax consequences to Will? No tax to Dan, will not include appreciation of property because death is not a realization event. (102-A) What basis does Will take in the land? No INCOME, but basis (step-up 1014-bequeath by death)= $250K. (g) What if the land had a FMV of $75K at the time of Dans death?

Basic Income Taxation B | 21

Basic Income Taxation Fall 2011 | Prof. McCormick No income, basis = Loss = $75K (1014step down, property went down during gift); gain = $100K (h) Assume Dan gives the land to his father, George, when the land has a FMV of $250K. George dies two months later and devises the land, still worth $250K, to Dans son Will. What basis does Will take in the land? $250K basis, but 1014(e) may disallow, but here its the son, so NOT applicable1014(a) FMV on day of deathbasis is $250K. Chapter 6: Sale of a Principal Residence Order of analysis 1.) a whole exclusion 1.)b amount (250k or higher amount?)(Flip up) 2.) Partial Exclusion 3.) Additional Limit for Sales in 2009 or after (GATPNQU) (121(b)(4) or (5) GATPNQU/Total Gain = Periods of non-qualified use/ total time owned. Exceptions---temporary change in employment/ military service 1. The IRC encourages home ownership. The exclusion under 121 applies to gains realized on the sale or exchange of ones principal residence. 2. Principal Residence: If a taxpayer alternates between 2 residences, the residence the taxpayer uses a majority of the time during the year will ordinarily be considered the taxpayers principal residence. Reg. 1.121-1(a)(2). See also Guinan case. 2. Majority of the time test Factors Relevant to determining a taxpayers principal residence: Reg. 1.121-1(a)(2). 2. Time is most important factor (See Guinan case). a. Short absences do not count against you. b. Look at each year and determine the # of months there! 3. Taxpayers place of employment 4. The principal place of abode of family members 5. The address listed on the taxpayers federal and state tax returns, drivers license, automobile registration, and voter registration card; 6. The taxpayers mailing address for bills and correspondence; 7. The location of the taxpayers banks 8. The location of religious organizations and recreational clubs with which the taxpayer is affiliated. 3. Ownership & Use Requirements: Under 121, taxpayers may exclude up to $250K ($500K with respect to certain joint returns) of the gain on the sale or exchange of a qualifying principal residence. 2. Does not require that the residence sold be ones principal residence at the time of the sale or exchange. 3. The statute requires only that the taxpayer have owned and used the property as a principal residence for periods aggregating two years or more during the five year period. The ownership and use requirements may be satisfied during nonconcurrent periods so long as the taxpayer satisfies each of them within the five year period ending on the date of the sale or exchange. REg 1.121-1(c).

a.

b.

a.

b.

Basic Income Taxation B | 22

Basic Income Taxation Fall 2011 | Prof. McCormick 2. Time period does not need to be consecutive, just must amount to a total of two years. a. Short, temporary absences, such as absences because of vacations or seasonal absence (even if accompanied by rental of the residence) will be counted as periods of use. c. Section 121 contains a number of special rules related to the ownership and use requirements of 121(a). 2. If an unmarried individual sells or exchanges property subsequent to the death of his or her spouse, the individuals use and ownership periods for purposes of Section 121(a) will include the period the deceased spouse owned and used the property. 121(d)(2). 3. If an individual receives property in a transaction described in 1041 (i.e. a transfer of property between spouses or former spouses), that individuals ownership period for purposes of 121(a) will include the ownership period of the transferor. 121(d)(3)(a); Reg. 1.21-4(b)(1). 4. If an individual continues to have an ownership interest in a residence but is not living in the residence because the individuals spouse or former spouse is granted use of the residence under a divorce or separation instrument, the individual will nonetheless be deemed to use property during the period her spouse or former spouse is granted the use of the property. 121(d)(3)(B); Reg 1.21-4(b)(2) 5. If an individual becomes physically or mentally incapable of self-care, the use rules are modified. a. If the individual owns and uses the residence for one year in the five year period, the individual will be treated as using the property for any period during the five year period in which the individual, while owning the property, resides in a facility satisfying certain requirements. 121(d)(7). 4. Amounts Excludable under 121: a. Many variables affect your analysis in a 121 Sale of a Principal Residence problem. They include: 2. whether only one home is sold; or 3. whether the taxpayer anticipates selling more than one home in a two-year period; 4. whether the taxpayer is single or married; a. if married, whether the taxpayer files separately or jointly; 5. whether the taxpayer is eligible for a whole exclusion under 121(a) or only a partial exclusion under 121(c); 6. and finally, if married and filing jointly, whether the taxpayer is eligible to increase his maximum exclusion from the $250K level to the $500K level. b. Whole Exclusion ( 121(a)) versus the Partial Exclusion of ( 121(c)) 2. Whole Exclusion - 121(a) a. Gain up to $250K (or $500K if husband and wife are filing jointly) from the sale of a principal residence is excluded from gross income. b. Requirements: i. 121(a): Property must have been owned and used by the taxpayer as the taxpayers principal residence for periods aggregating 2 years or more during the 5 year period ending on the date of the sale/exchange. 1. So, seller must have owned the property for 2 years out of the 5 years leading up to the sale 2. AND seller must have used the property as a principal residence for 2 years out of the 5 years leading up to the sale. ii. 121(b)(3): Seller cannot use 121(a) if seller used 121(a) or (c) within 2 years before the sale. 1. So, if seller used the 121 exclusion within the 2 years previous to this sale, then he or she cannot use the whole exclusion -- 121(a) again now. c. What is the amount of the exclusion?

Basic Income Taxation B | 23

Basic Income Taxation Fall 2011 | Prof. McCormick Either $250K or $500K. 1. The maximum exclusion available is $250K. However, if the taxpayer is married and files a joint return, then that $250K may flip-up to a $500K maximum exclusion amount. ii. There are 2 flip-up rules: 1. The Regular Flip-Up Rule: 121(b)(2)(A). a. The maximum exclusion becomes $500K for a husband and wife filing jointly if: a. The taxpayer is married and files jointly AND b. Either spouse owns the property for the 2 years (out of the 5 years leading up to the sale) AND c. Both spouses use the property as a principal residence for the 2 years (out of the 5 years leading up to the sale) AND d. Neither spouse has used the 121 provision within 2 years earlier. 2. Special Joint Filing Flip-Up Rule: 121(b)(2)(B) a. If the jointly filing spouses do not meet the requirements of the regular flip-up rule, then the exclusion limitation shall be the sum of the two exclusions the spouses would get computed as if they were each single and as if each is treated as owning the property during the period that either owned it. c. Partial Exclusion - 121(c) 2. If the taxpayer is not eligible for the whole exclusion under 121(a) (either $250K or $500K) because he or she fails one of the three 2-year requirements, then check to see whether the partial exclusion under 121(c) is available. 3. Eligibility: a. The partial exclusion is available only if the same was necessitated by a change in place of employment, health, or unforeseen circumstances listed in regulations or other guidance. 4. Amount of Partial Exclusion: a. Formula for Partial Exclusion as per 121(c): 5. Shorter of: Amount to be excluded = [time owned & used] or [time since the last 121(a) sale $250,000 OR $500,000 2 years 6. In the Regs this equation is: Treas Reg 1.121-3(g)(1): Amount to be excluded = [time owned in last 5 years], [time used in last 5 years] or [time since last 121 sale] $250,000 OR $500,000 2 years (use # of days or months). a. Even though the regulation is inconsistent with the Code, it is unlikely that the regulation will ever be challenged by a taxpayer since it gives a more generous result a larger partial exclusion in many cases than does the code. C. Additional limit on exclusion for homes sold after Jan. 1, 2009. 121(b)(4)/(5) i.

1. For sale of houses after Jan. 1, 2009, you must also consider a separate limit on the amount of excludable gain: 121(b)(4) or (5). For houses sold after that date, any gain allocated to a period of nonqualified use (a period taxpayer did not use the house as a principal residence with some exceptions) is not excluded.

Basic Income Taxation B | 24

Basic Income Taxation Fall 2011 | Prof. McCormick a. Formula: Gain allocated to period of nonqualified use = periods of nonqd use Total Gain Period owned b. Military service does not count as nonqualified use! 2. Election: 121(f) a. If you expect to sell a second house within the next two years for a large gain, you may not want to use your 121(a) or (c) exclusion on the sale of a first house. Why not? Doing so may preclude you from using the exclusion again for the second sale in the next two years. To allow flexibility, Congress gave taxpayers the option of electing not to use the exclusion on the first sale. 121(f). You make this election simply by reporting and including the gain in gross income. See also, Reg 1.121-4(h).

Chapter 6 Problems: 1.) B & J bought a home in Seattle in 2001 for $350K, joint tenants with right to survivorship. It was their principal residence until May 2006. They moved to Northern Idaho and bought a home for $250K. In Jan. 2007, B & J sold their Seattle home. The purchaser paid B and J $600K in cash and assumed a $250K mortgage encumbering the property. B and Js adjusted basis in the house was $400K. a) Explain the tax consequences to B & J on the sale of their Seattle home. Assume that B & J filed a joint tax return for that year. B & J file joint tax return. Amount realized = $850K (600+250); Adjusted B = $400K. Gain = AR AB = $450K. The question now becomes, do B and J meet the requirement in 121? There are 3 two-year requirements: 1. ownership must own property for 2/5 years prior to sale, 2. Use as principal residence for 2/5 years prior to sale, and 3. Have not used 121 in the last 2 years. In this case, they are eligible for the whole exclusion. They can at least exclude $250K, but they may use the flip-up exclusion to increase their exclusion from $250K to $500K if they meet the requirements in 121(b)(2)(A) (1) both spouses file joint tax return, (2) either spouse must own the property at least 2 years, (3) both spouses must use the property as their principal residence for at least 2 years and (4) neither spouse has used exclusion under 121 before in the last two years. In this case, B & J meet the requirements and they are eligible to exclude the gain from the sale up to $500K. b) Would the answer to (a) change if the title to the Seattle home was held by J, in her name alone? It would not make a difference in the result, but our analysis, we only focus on the taxpayer and determine whether J satisfies the requirements in 121(a), so she is entitled to the whole exclusion. 2.) Jan 1, 2008 Chris moved out of her condo in Miami. Fort Lauderdale was Chris principal residence for all of 2008. Tom purchased it for $600K on June 30, 2005 and

Basic Income Taxation B | 25

Basic Income Taxation Fall 2011 | Prof. McCormick moved into it that day. This remained a PR through 12/31/2008. Jan 1, 2009, Chris and Tim rented an apartment in Key West and moved in that day. Key West apartment was principal residence for both of them for all of 2009. Fort Lauderdale house remained vacant through 2009. June 30, 2009 Chris sold her Miami condo for a gain of $200K, which she excluded from income under IRC 121. Jan. 1, 2010 Chris married Tom and they moved back to Fort Lauderdale that day. Fort Lauderdale house was their principal residence for the first 6-months of 2010. Tom sold the Fort Lauderdale house, titled in his name for $1M. July 1, 2010 = Chris and Tom moved to San Francisco. Chris got a promotion from her employer requiring a move. (a) Can Chris & Tom exclude gain on the sale of the Fort Lauderdale house? Gain = $1M - $600K = $400K. How much can be excluded? -Whole Exclusion? (a) ownership June 2005-07/2010 = YES! (b) use 3.5 years + .5 years = YES! (c) not used 121 before = YES! Can take whole exclusion despite Chris use because Tom never used 121. Flip-up? Cant use because Chris didnt live there for at least 2/5 years before the sale. Use special flip-up rules: (treat as if each is single) Tom = full exclusion of $250K Chris = part because she doesnt meet the use requirements used in 121 exception. 121(c): moved for change in job Shortest of: time C owned time C used as PR time since last used 121 X = ______________________ $250,000 2 years ____X________ = 1 year = $125,000 exclusion $250,000 2 Total Exclusion for C & T = $250K (tom) + $125K (Chris) = $375K; report $25K as income. (325 is excluded under 121 only 25 has to be reported as income) Separate rule is requires 80k to be excluded so have to take the larger----------on the board she had 400k 121(C)-----Chris Exclusion if single/250k = shortest of time C owned; time C used as PR; tiem since last used 121/2years

Basic Income Taxation B | 26

Basic Income Taxation Fall 2011 | Prof. McCormick (What her whole exclusion would be if she were single) (Partial Exclusion is X/250k = 1year/2y ------ x= 125k Remember the rule of 80k

(b) What if Tom and Chris were a gay couple? Cant file a joint return (they werent married). Tom still gets whole exclusion, so $150K of income.

3.) Start with 121(b)(1)-(2) A, who is single, sells a home he owns in Montana for $400K. His adjusted basis in the home is $200K. During the past 5 years, A lived in the Montana home 5 months each year. For the first 3 years: 4 months per year in an apartment in Portland; 3 months per year in a cottage he owns on the Oregon coast. Most recent two years = lived 7 months per year in the cottage and did not live in the apartment at all. Considering selling the Montana home and the Oregon cottage. Can he use 121(a)? Oregon 9 months, 14 months = 23 months 2 years Montana 5 months, 10 months =15 months 2 years PR in Y4&Y5 is the Oregon home MN home is not PR because he didnt spend majority of time

** Potentially could use either residence as PR (argue factors), but to sell both hed have to choose which hed want to sell. Chapter 9: Discharge of Indebtedness Income 1. Does the forgiveness of the loan generate income? 2. If a taxpayer were insolvent before and after the discharge or cancellation of a debt, no income resulted. Dallas Transfer & Terminal Warehouse Co. 1. Relies on the concept that there is a reduction or extinguishment of liabilities without any increase of assets. ii. If, as a result of the discharge, the taxpayer is again solvent, there is income and the debtor realized income to the extent that the discharge of indebtedness made the debtor solvent. 1. Insolvency Exception: no income arises from discharge of indebtedness if the debtor is insolvent both before and after the transaction; and if the transaction leaves the debtors with assets whose value exceeds remaining liabilities (solvent), income is realized only to that extent.

Basic Income Taxation B | 27

Basic Income Taxation Fall 2011 | Prof. McCormick 2. Provided by 108(a), but its not perfect, 108(b) takes away---it requires you to reduce one of you tax attributes. 108 provides that the discharge of indebtedness will not generate gross income if the discharge occurs in a bankruptcy case or if the discharge occurs when the taxpayer is insolvent. 108(a)(3) limits the insolvency exclusion to the amount by which the taxpayer is insolvent. 1. Insolvent means the excess of liabilities over the fair market value of assets. The determination is based on the basis of the taxpayers assets and liabilities immediately before the discharge. a. Amount of Insolvency = Asset Indebtedness Other Liabilities i. With respect to other liabilities in determining insolvency and for the benefit of the insolvency exception, the taxpayer must prove with respect to any obligation claimed to be a liability, that, as of the insolvency calculation date, it is more probable than not that he will be called upon to pay that obligation in the amount claimed. 1. This is common sense. If you likely will not have to pay back the other liabilities, then you should not receive the benefit of claiming them against your current insolvency calculation. b. Amount Realized from Discharge of Indebtedness = Indebtedness Setlment c. Net Income Recognized = AR from the discharge Amount of Insolvency (prior to settlement) Disputed or Contested Debts (Another EXCEPTION) 1. If the amount of a debt is disputed, settlement of the amount does not constitute a discharge of indebtedness. a. Disputed Debt/Contested Liability Doctrine provides that a subsequent settlement of that dispute is treated as the amount of debt recognizable for tax purposes. The excess of the original debt over the amount determined to have been due may be disregarded in calculating gross income. i. This is based on good faith dispute of the amount of debt. 1. See Preslar v. Commissioner. 2. Or Zarin a liquidated debt, if unenforceable, would implicate the contested liability doctrine. Purchase-Money Debt Reduction for Solvent Debtors 1. 108(3)(5) provides that if a taxpayer agrees to pay the purchase price, subsequently refuses to pay the entire balance and the parties resolve their dispute by agreeing to a reduction in the purchase price, no income resulted but rather merely a retroactive reduction in the purchase price. a. The basis of the taxpayer in the property was correspondingly reduced as well. Acquisition of Indebtedness by Person Related to Debtor

iii.

iv.

v.

vi.

Basic Income Taxation B | 28

Basic Income Taxation Fall 2011 | Prof. McCormick 1. For purposes of determining how much income a taxpayer has from the discharge of indebtedness, 108(e)(4) specifically provides that if a person related to a debtor acquires the indebtedness, the acquisition shall be treated as an acquisition by the debtor. Therefore, the debtor will be prevented from avoiding discharge of indebtedness income. Discharge of Deductible Debt 1. 108(e)(2) provides that forgiveness of a debt does not generate income if the payment of the debt would have been deductible. a. The rationale for this provision is that, if discharge of indebtedness income were to be imputed to the tenant, the tenant would be entitled to a deduction for the past due rent which would completely offset the discharge of indebtedness income. Discharge of Certain Student Loans 1. 108(f)(1) excludes gross income amounts otherwise includable in gross income by reason of the discharge of any student loan. 2. Rev. Ruling 1008-34, concluded that the 108(f) exclusion applied to the discharge of loans made under a law schools Loan Repayment Assistance Program (LRAP). Discharge of Qualified Principal Residence Indebtedness 1. For the unfortunate taxpayer/borrower who lost her home another problem loomed: the amount of forgiven indebtedness would normally constitute discharge of indebtedness income under 61(a)(12). In response to the foreclosure problem, Congress enacted 108(a)(1)(E) and (h), providing an exclusion for qualified principal residence indebtedness discharged on or after January 1, 2007 and before January 1, 2013. a. Applicable to the discharge of any qualified principal residence indebtedness so long as the discharge is directly related to a decline in the value of the residence or to the financial condition of the taxpayer. 2. Qualified Principal Residence Indebtedness is defined as up to $2 Million of indebtedness secured by the taxpayers principal residence so long as the indebtedness is acquisition indebtedness, i.e., indebtedness incurred in constructing, acquiring, or substantially improving the residence. a. 108(h)(5) states that the principal residence shall have the same meaning as it does for purposes of the 121 exclusion. 3. A taxpayer taking advantage of the exclusion under 108(a)(1)(E) must reduce (but not below zero) her basis in her principal residence by the amount excluded. Discharge of Indebtedness as Gift, Compensation, Etc. 1. Commissioner v. Jacobson the court considered whether a discharge of indebtedness could be considered an excludable gift under 102(a) and its predecessors. a. Concluded the gift exclusion was not applicable where a debtor purchased his own obligations at a discount. 2. However, in certain contexts, the cancellation of indebtedness can be an excludable gift. If a parent lends money to a child and then

vii.

viii.

ix.

x.

Basic Income Taxation B | 29

Basic Income Taxation Fall 2011 | Prof. McCormick subsequently forgives the debt, the forgiveness of the debt would likely be considered a gift excludable under 102(a). a. Use the Duberstein test, look to donative intent. 3. If the cancellation of debt is a means for the employer to compensate you for services performed, then it will be considered income.

Chapter 9 Problems: 1.) Donald borrowed $5K three years ago to assist his daughter who had incurred significant legal fees in a divorce battle. After repaying $3K of the loan, Donald was forced to resign from his job for health reasons and while he was not technically insolvent, Donald was unable to continue to make payments on the loan. What tax consequences to Donald if the lender were: (a) a local bank? No exception, $2K will be income to Donald because he was not insolvent. (b) Donalds employer? If from employer, either compensation for services or cancellation of indebtedness income. (102) (C ) (1)---if employer then it wont be considered a gift. (c) Donalds brother? Must look at the intent of the brother. If gift, then no income. 2.) Kevin received a bill for $10K for landscaping work done on his personal residence. Kevin, who was unhappy with the quality of the work, refused to pay the $10K. The landscaper reduced the bill to $6K and Kevin paid that amount. (a) Assuming Kevin is solvent, does Kevin have discharge of indebtedness income under these circumstances? No income. See the contested liability doctrine and Preslar & Zarin. Just renegotiating the purchase price. If liquidated (amount of debt is known), then the debt is cancelled, not renegotiated and must pay income. 10th Circuit says that it cannot be used when liquidated because you agreed to the debt amount. If in 3rd Circuit: can use the contested liability doctrine, even if liquidated. If debt is unliquidated then the taxpayer can use the contested liability doctrine. (b) How would your answer to (a) change, if at all, if the landscaper first succeeded in getting a judgment in against Kevin for the full $10K and only thereafter agreed to the $6K payment in settlement of the dispute when Kevin threatened to appeal the judgment? Need a final unappealable decision to be income. Use renegotiated debt. (c) How would your answer to (a) change, if at all, if the landscaping work had been done on Kevins business premises, Kevin did not dispute the amount owed but the landscaper,

Basic Income Taxation B | 30

Basic Income Taxation Fall 2011 | Prof. McCormick because of Kevins cash flow problems, was willing to accept a $6K payment in satisfaction of the $10K bill. In answering this question, assume Kevin would be entitled to deduct as a business expense the cost of the landscaping work. No income, deductible as business expense under 108(e)(2).

3.)

In connection with her business, Samantha borrowed $50K from Lender last year, giving Lender an unsecured note in that amount. This year, because of a deteriorating economic climate, Lender agreed to accept $25K in cash from Samantha in full settlement of the note on which Samantha still wed $45K. (a) Does Samantha have discharge of indebtedness income? If insolvent, no income under 108(b)(1)(B). If solvent, then income $20K of debt is cancelled. (b) Would your answer change if, instead of Samantha paying Lender $25K, Samanthas parents purchased Samanthas note from Lender for $25K and then promptly forgave Samantha the amount owing under the note? Under 108(e)(4)(a), if a related person to debtor acquiring debt from unrelated party. Acquisition of debt is treated as if Samantha acquired the debt cancellation, $20K of income. No income to Samantha. ( I think) Look at 61(a)(12)

4.)

As a wedding gift for his dauther, Louis purchased a 1921 Steinway concert grand piano from Sharon for $50K, giving Sharon $10K in cash and agreeing to pay her the balance over a five-year period. This year, when Louis still owed Sharon $25K, Louis informed Sharon that, because of market losses, he would have a problem paying her the full balance owing on the piano. Sharon agreed to reduce the amount owing by $10K. What tax consequences, if any, to Louis? Under 108(e)(5), this is a purchase price adjustment. Must assume solvency and since all conditions are met, no income because price of item was renegotiated. Basis in the piano will be lowered to $40K because he must reduce a tax asset, income is just deferred until the sale.

Debt has to be owed to owner of property, purchaser cannot be insolvent, the cancelled debt would count as income. Can use the above section, and allows the reduction of debt to be a purchase price adjustment. 5.) In 2005, Rhonda purchased a home in Nevada for $500K and made that home her principal residence. She borrowed $400K of the purchase price for the home from a national lender. This year, because of a dramatic economic downturn, Rhondas home had a FMV of only $250K. Rhonda, who still owed $370K on the purchase money mortgage encumbering her home and was struggling to make the monthly mortgage payments, entered into a loan restructuring agreement with the lender. Under their

Basic Income Taxation B | 31

Basic Income Taxation Fall 2011 | Prof. McCormick agreement, the lender forgave $150K of the loan and Rhondas monthly payments were recalculated accordingly. (a) Assuming Rhonda is solvent, what tax consequences to Rhonda as a result of the forgiveness of $150K of the debt on her home? (Look to bottom of page 16in regulation) Discharge of qualified principal residence, 108(a)(1)(E), but must reduce the basis in the home by $150K. 108(h)(3) price in home dropped and shes struggling to pay the mortgage. 108(h)(2) defines what a qualified principal residence is. If meet all requirements, then no income, just reduce the basis. (b) Assume, instead of entering into a loan restructuring agreement, Rhonda simply stopped making payments on the loan, the lender foreclosed on the mortgage and, at a foreclosure sale, the lender sold the home for $250K. In addition, assume the lender on the purchase money mortgage. What tax consequences, if any, to Rhonda? $120K forgiven not income, reduced basis though. 108(B)(1)(e) applies. This yields a personal loss, unable to deduct! (c) How would your answer to (a) change if, instead of the Las Vegas home being Rhondas principal residence, it was Rhondas vacation home? If it is a vacation home, then income because the home is not a qualified principal residence. 6.) Bill borrowed $200K from Judy and later, when Bill was insolvent, Judy accepted a tract of unimproved land from Bill in satisfaction of the debt. The land, which Bill had purchased as an investment for $50K, had an appraised FMV of $150K at the time Judy received it. Immediately prior to the transaction, Bills liabilities included the $200K debt to Judy and $50K of indebtedness to other parties. In addition, however, Bill had guaranteed repayment of a $25K bank loan his son had taken out. The loan comes due in the near future, and the son estimates that there is a 50/50 change he can repay it. Other than the tract of land Bill transferred to Judy, Bills only other assets are his personal property including his cars and household furnishings, which have a total value of $75K. As a result of Judys acceptance of the land in satisfaction of the debt, these items of personal property are not Bills only assets. He continues to owe $50K to third parties and to be the guarantor of his sons $25K loan. (a) How much discharge of indebtedness income, if any, must Bill report as a result of the settlement of the debt owed to Judy? Does he have any other income? $50K of debt was cancelled. See Merkle case if > 50% chance that he would have to pay the debt, than it counts as his own debt!!! Sons debt here does not count as his own because its not > 50%. Determine if insolvent: BEFORE AFTER

Basic Income Taxation B | 32

Basic Income Taxation Fall 2011 | Prof. McCormick Debts: $200K $50K Total: $250K Assets: $150K Land $75K personal $225K total *** Insolvent by $25K $50K $50K $75K

** Solvent by $25K

Can exclude to the extent that he is insolvent, $25K. $25K is income through 108(a)(3). Can use insolvency exception, must reduce basis in taxable asset. Like a car, assume one asset to reduce basis in, wont ask about allocation across multiple assets. Gain on $100K on the laon for the land (b) What basis will Judy take in the land she received from Bill? Judy will take the basis of the FMV of the land. $150K 7.) Lloyd is a professional actor. At a time when his liabilities exceeded the FMV of his assets, Lloyd received $10K for his appearance in a toothpaste commercial. Even after receiving this money, Lloyd remained insolvent. What are the tax consequences to Lloyd of receiving the $10K? Does your answer change if Lloyd owed $10K to the producer of the commercial and the producer canceled this debt in lieu of paying $10K to Lloyd? Lloyd has income of $10K for compensation for services. If cancelled debt, then no, hes being compensated for service, even though hes got debt. Cant use because hes actually being paid for work.

Chapter 10: Compensation for Personal Injury & Sickness 1. Can these damages be excluded? Is there a theory that will allow exclusion? a. Theories: i. (1) 104(a)(2) ii. (2) Raytheon case 2. 104 & 105 exclude from gross income certain amounts received on account of personal physical injury or sickness. This is called an expression of Congressional compassion for those who suffer personal physical injury or illness. 3. Damages a. Business or Property Damages i. No physical injury here, most likely not able to be excluded! ii. Unless there is a specific rule to the contrary, damages awarded on account of lost profits would be taxable; a recovery of property damage would be measured against the basis of the Property to determine the taxpayers realized gain or loss.

Basic Income Taxation B | 33

Basic Income Taxation Fall 2011 | Prof. McCormick 1. See Raytheon Products Corp v. Commissioner, instructing that we ask in lieu of what were the damages awarded. a. Compensation for the loss of goodwill in excess of its cost is gross income. Look at the thing being replaced, if it would be treated as income, then it should be treated the same. b. Damages Received on Account of Personal Physical Injuries or Sickness i. If damages fall under 104(a)(2), then the damages that are excluded are very broad. Once you fall within the section, dont use the Raytheon rule, apply the statutory language. ii. 104(a)(2) excludes from income any damages received, whether by suit or agreement, as a lump-sum or periodic payment, on account of personal physical injuries or physical sickness. 1. Distinguishes between personal physical injuries and all other injuries, including non-physical injuries and injuries to ones business or property. 2. In Threlkeld v. Commissioner, the court held that personal injury for the purposes of 104(a)(2) referred to any invasion of the rights that an individual is granted by virtue of being a person in the sight of the law. a. This was soon changed! c. Supreme Court Limitations on the Pre-1996 Version of 104(a)(2) i. Burke sexual discrimination damages must be included as income. ii. Commissioner v. Schleier, stating that the key question to ask is whether the damages received were on account of (actually compensated for) personal injury. 1. This indicates that damages are on account of personal injuries for 104(a)(2) purposes only if they bear a close nexus to the personal injury, or the damages are intended to compensate the taxpayer for the personal injury and the consequences casually linked to the injury. a. If that relationship between the damages and personal injuries does not exist, no exclusion is available. d. The 1996 Amendments to 104 i. Congress chose to limit the exclusion by restricting it to those damages received on account of physical injuries or physical sickness. ii. Emotional distress is not to be treated, save only for related medical care expenses, as a physical injury or physical sickness. 1. Look to p. 100-101 of the Code Book, stating that emotional distress damages will be included in gross income. 2. The term emotional distress includes physical symptoms stemming from the emotional distress, but the medical care for the physical symptoms, will be excludable. a. If damages for physical symptoms, then look to the origin of the claim. If the claim is physical, then excluded, but if not, then no! iii. Pain and suffering will always be excluded. 1. The jury must be told whether the damages will be taxable or not. iv. To determine excludability:

Basic Income Taxation B | 34

Basic Income Taxation Fall 2011 | Prof. McCormick 1. Origin-of-the-Claim notions, in conjunction with the rule that emotional distress is not a physical injury or sickness. a. If an action has its origin in a physical injury or physical sickness, whether or not the recipient of the damages is the injured party, then damages are excluded from income. b. If action is not from a physical injury or physical sickness, then 104(a)(2) does not exclude damages. It would count as income. i. I.E. Sexual discrimination, would not count. 2. The exclusion from gross income does not apply to any damages received (other than for medical expenses) based on a claim of employment discrimination or injury to reputation accompanied by a claim for emotional distress. a. (1) if the claim has its origin in a personal physical injury, a recovery for emotional distress may be excludable. b. (2) if the claim has its origin in a physical injury, it is not necessary that the recipient of the damages is the individual who suffered the physical injury. 3. Private letter ruling: direct or unwanted or uninvited physical contacts resulting in observable bodily harms, such as bruises, cuts, swelling, and bleeding are personal physical injuries under 104(a)(2). a. Physical restraint and physical detention pursuant to arrest, including being handcuffed and searched and deprived of personal freedom, are not physical injuries for purposes of 104(a)(3). 4. These changes place enormous weight on whether the origin of a claim lies in a physical injury, and deny this all-important physical injury status to a significant, but undefined range of physical symptoms grouped under the term emotional distress. e. Punitive Damages i. Punitive damages are not covered under 104(a)(2) because they are an element of damages not designed to compensate victims, rather they are punitive in nature. ii. Statutory exclusion does not apply to punitive damages, they count as income!!! f. Allocation of Awards i. The IRS will scrutinize settlement carefully and will dispute allocations resulting from settlement negotiation, which the IRS does not consider to be arms length. 1. The Tax Court could make its own determination of the proper allocation of the settlement proceeds. ii. Robinson v. Commissioner The Tax Court held it was not bound by a state court judgment allocating 95% of certain settlement proceeds to tort-like personal injuries. iii. Bagley v. Commissioner the court agreed the services should not be bound by a settlement agreement allocating all of the $1.5 million settlement to personal injuries.

Basic Income Taxation B | 35

Basic Income Taxation Fall 2011 | Prof. McCormick 1. The court emphasized that the critical question in determining the tax status of settlement amounts was in leiu of what was the settlement amount paid. iv. McKay v. Commissioner the Tax Court respected an allocation in the settlement agreement of approximately of the settlement award to personal injury claims and to taxpayers contract claims. v. Standard for who prevails: look at the pleadings to determine what is reasonable. g. Periodic Payments i. Excludable under 104(a)(2). ii. Lost wages resulting from physical injury will be covered by 104(a)(2) and will be excluded from income. 4. Accident and Health Insurance a. Under 104(a)(3), payments received through accident or health insurance policies are excluded from gross income, provided the policy was not financed by the taxpayers employer or by employer contributions not includable in the taxpayers income. b. If employee paid = it is excluded under 104(a)(3) c. Payments made by employer-financed accident and health plans are not exempt under 104(a)(3), but instead are governed by 105. i. 105 generally includes these payments in the employees gross income. 1. Exceptions: a. For medical expense reimbursements OR b. Certain payments for permanent bodily injury or disfigurement. i. Actual medical expenses incurred. d. 106(a) permits employer contributions to accident and health plans to be made on a tax-free basis to the employee, but 105(a) makes payments under such employer-financed plans taxable, expect to the extent 105(b) or (c) applies. e. Payments under employer-financed plans are excluded to the extent they compensate for permanent bodily injury or disfigurement of the taxpayer, spouse, or disfigurement of the taxpayer, spouse, or dependents, provided the payments are computed with reference to the nature of the injury and not the period of absence from work. 105(c). i. If such amounts are paid under workers compensation act, they are excluded by 104(a)(1). Reg. 1.105-3. 5. Previously Deducted Medical Expenses a. 213 permits a deduction for unreimbursed medical expenses in excess of a floor based on the taxpayers adjusted gross income. b. Medical expenses incurred on account of a personal physical injury, unreimbursed and thus deducted in the year paid, may be reimbursed in a later year as part of a claim for damages on account of personal physical injury. i. Amounts attributable (despite the nexus) to previously deducted medical expenses are not excluded from gross income. ii. Reimbursements for nondeductible medical expenses, however, are excluded from income. c. In some instances, a payment for personal injury or sickness, based in part on the taxpayers medical expenses, may be made in the form of an undifferentiated lump sum.

Basic Income Taxation B | 36

Basic Income Taxation Fall 2011 | Prof. McCormick d. In Rev. Ruling 75-230, 1975, the Service required that a lump sum award in a personal injury suit settled out of court be allocated between medical expenses and other components of the award. 6. Workers Compensation a. 104(a)(1) excludes from income amounts received under workers compensation acts as compensation for personal injuries or sickness. 7. Certain Disability Pensions a. Military disability pensions and certain other government disability pensions are excluded from income under 104(a)(4). i. 104(b) limited to persons receiving compensation for combat-related injuries and to those who would on application receive disability compensation from the Veterans Administration. ii. A special provision also provides an exclusion for disability income attributable to injuries suffered in a terrorist attack upon an employee of the US engaged in performance of official duties outside the US. 8. Hypotheticals for Clarification: a. Guy jumped out of bushes and assaulted your client by hitting your client in the head. You sue wrongdoer. i. Suing for A&B, Emotional Distress, hair is falling out because of emotional distress, and medical treatment for hair falling out and ulcer. 1. Are damages excluded? a. Yes. All damages for A&B are excluded. b. Emotional distress are also excluded. Look to the Origin of the claim physical injury c. The symptoms are physical personal injuries and 104(a)(2) applies, excluded from gross income. d. Medical expense damages are also excluded. b. Client was sexually harassed at work. Sue employer for sexual harassment, emotional distress, physical symptoms stemming from emotional distress, and medical expenses from treatment of symptoms. i. Sexual harassment damages are not able to be excluded. ii. Emotional distress damages are not excluded. iii. Physical symptoms stemming from emotional distress not excluded. iv. Medical expense damages are able to be excluded.

Chapter 10 Problems: 1.) The building that Emily conducted her travel agency was destroyed in a fire. Emily decided not to resume her business. In the negligence action, she brought against both the shopping mall owners and the company they hired to produce the fireworks display, Emily recovered $350K for the destruction of the building and $150K in lost profits. Emily had an adjusted basis of $200K in the building, which had a FMV of $350K. What are the tax consequences to Emily? $150K $350K Lost-profits are taxed. See Raytheon case, replacing something that would have been income, and thus they count as income. Building . Her basis was $200K $350K-$200K= $150K in gain. $150K must also be taxed.

Basic Income Taxation B | 37

Basic Income Taxation Fall 2011 | Prof. McCormick $300K of taxable income; $200K is excluded because of her basis in the building. 2.) Tom, a self-employed landscape architect, was injured last year in an accident in which he was struck by a car driven by a drunk driver. Tom filed a negligence suit against the driver, seeking over $2 Million in compensatory and punitive damages. This year, the parties to the action settled the suit out of court for $900K, which the parties agreed to allocate as follows: Payment for Pain and Suffering: Reimbursed Medical Expenses: Future Medical Expenses: Lost Income: Punitive Damages: $500K $100K $50K $100K $150K

Tom paid the medical expenses of $100K last year from his own funds and was allowed a medical expense deduction of $70K under 213. (a) What are the tax consequences to Tom as a result of this settlement? In answering the question, consider each item to which the settlement was allocated. P&S: excludable because of a personal physical injury, 104(a)(2) applies. ME: $30K excludable, because he already used the 213 deduction. 104(a)(2) states this. FME: $50K is excludable, if he excludes now, he cant deduct later when he incurs the medical costs. LI: Excluded under 104(a)(2); Raytheon doesnt apply here. PD: Exclusion does not apply for punitive damages. (b) Assume Tom insisted that the entire $900K settlement be allocated to pain and suffering even though in his lawsuit he had sought recovery for the items listed in the settlement described above. Desiring to settle the lawsuit, the defendants agree to the allocation. What are the tax consequences to Tom of this settlement? Tom is better off allocating entire settlement to P&S, where it is an excluded category.

(c) What are the tax consequences to Toms wife Phyllis if she recovers $100K from the defendants for loss of consortium due to the injuries suffered by her spouse? How would your answer change, if at all, if Phyllis accepts the offer of defendants to pay her a total of $150K if she will agree to payment of that amount over a five year period in equal annual installments of $30K each? Does any theory exist to exclude from income from Phyllis? 104(a)(2) because it is her spouses injury, doesnt necessarily have to be same person that is receiving damages as the person who was injured. She can exclude $100K.

Basic Income Taxation B | 38

Basic Income Taxation Fall 2011 | Prof. McCormick What if she accepts $150K if she agrees to installments of $30K each? Still excluded, 104(a)(2) allows for lump-sum or periodic payments. 3.) Martha, a business graduate of a major university and a CPA, accepted a position with a prominent accounting firm. Shortly after Martha complained to a partner of the firm that she was being sexually harassed by a manager of the firm, she was fired. As a result of the sexual harassment and her discharge, Martha suffered severe emotional distress which exacerbated a pre-existing ulcer condition and ultimately resulted in her hospitalization for bleeding ulcers. Martha suffered depression and is being treated by a psychologist. Martha has been unable to locate suitable employment. In full settlement of the claims, the firm offered Martha $500K. Martha would owe her attorney $150K. In assessing whether to accept the settlement, Martha asked you to advise her regarding the potential tax consequences of the settlement. What advice would you give? Categories of Damages: Emotional Distress: not the result of physical injury, not excludable. Sexual Harassment: not excludable, not physical. Wrongful Discharge: not under 104(a)(2), not physical Exacerbation of the Ulcer wasnt physical injury cant claim. Treatment for bleeding ulcer as long as not deducted, medical expenses = excluded Depression not excludable look to origin of claim Treatment for depression excludable because it is a med. Expense. Lawyers fees: usually get a deduction. Made the deduction 62(a)(2) creating an above the line deduction as long as the award counts as income and when the case involves civil rights enforcement or employment discrimination. So here, shed receive this deduction! Would your advice be any different if Martha could establish that the sexual harassment she experienced at the firm involved not only verbal statements and gestures by the manager but also unwelcome physical contact, including embraces and kisses? No, not necessarily a physical injury. What difference, if any, would it make if in one instance the manager grabbed Martha by the arm and actually bruised her arm when she tried to break away from him? There must be observable harms: Private letter ruling: direct or unwanted or uninvited physical contacts resulting in observable bodily harms, such as bruises, cuts, swelling, and bleeding are personal physical injuries under 104(a)(2). 4.) Susan was injured in an accident and lost the sight in one eye. She incurred $30K in medical expenses. She had an accident and health policy she had purchased with her own funds that paid her $15K for medical expenses, $10K for lost wages, and $25K for the loss of sight. What tax consequences from her receipt of these payments? $15K for ME: $10K for LW: $25K loss of sight:

104(a)(3) applies, and all are excluded!

Basic Income Taxation B | 39

Basic Income Taxation Fall 2011 | Prof. McCormick Alternatively, assume that her only policy was an accident and health policy provided by her employer which paid her $20K for medical expenses, $12K for lost wages, and $20K for the loss of sight. What tax consequences from her receipt of these payments? Since purchased by her employer, it falls under 105, even though theyre not income under 106. $20K ME: excludable under 105(b), if look at regulations, that exclusion is limited to actual medical expenses. The excess reimbursement will count as income! Not applicable under 104(a)(3). taxable, replacing something that would be taxable! excludable under exception for loss of limb 105(c).

$12K LW: $20K Sight:

Finally, assume that Susan had both the policy she purchased and the policy her employer provided and that the two policies paid the amounts described above. What tax results? Gets both sets of payments: Self-financed Policy 104(a)(3) Employer Financed Policy 105 (b)&(c)

actual medical expenses were $30,000. The policy giving $20K in medical expenses, over the $15K, that $5K would be income and taxable. All three in her policy is still not-taxable. The $20K will be excluded still ( 105(c)), the $12K is included in income under 105(a), and then the excess of reimbursed medical expenses, $5K will be income. Look at it proportionally: $15K from self-financed plan; $20K from eer plan 43% of total reimbursement is $15K, 57% of total reimbursement is the $20K Then use the $5K by 43%, thats how much is not taxed! The rest is then taxed! $2850 will be taxed.

Chapter 14: Depreciation 1. To enable taxpayers to deduct the costs associated with the use of business or investment property, Congress developed a cost recovery system allowing taxpayers to write off or deduct their capital investment (or cost) over a specified period of time the recovery period. 2. Depreciable Property a. 167, the principal depreciation provision, defines depreciation deduction as a reasonable allowance for the exhaustion, wear and tear (including a reasonable allowance for obsolescence) (1) of property used in the trade or business or (2) of property held for the production of income. b. One may not depreciate a personal residence. Cannot depreciate personal exceptions. c. Requirements: i. The property must be (1) used in ones trade, business, or for investment it must also be (2) subject to wear and tear, decay or decline from natural

Basic Income Taxation B | 40

Basic Income Taxation Fall 2011 | Prof. McCormick causes, exhaustion, or obsolescence. Land, stocks, and other assets are not depreciable. Recovery Period The Useful Life Concept a. 168(e)(1) defines classes of property. The 7-year property class is now the catch-all class and includes all personal property with no assigned classlife under the ADR (Asset Depreciation Range). Depreciation Methods a. The Code utilizes both straight line depreciation and accelerated depreciation methods. 168(b)(1) provides that, with respect to 3-year, 5-year, and 7-year property, the 200% declining balance method shall be used, but the taxpayer shall shift to the straight line method in the year that method would provide the larger deduction. b. 168(b)(3) requires taxpayers to use the straight line method to depreciate residential rental property and nonresidential real property. Conventions a. The applicable recovery period begins when the property is placed in service. Place in service means placed in a condition or state of readiness and availability for the specifically assigned function. b. Residential rental property and nonresidential real property placed in service during any month are deemed placed in service on the mid-point of such month. 168(d)(2), (4)(B). c. No depreciation deduction is allowed for property placed in service and disposed of during the same taxable year. d. The other classes of property are generally subject to a half-year convention any property placed in service during the tax year is deemed placed in service on the mid-point of the tax year. 168(d)(1) and (4)(A). e. If the properties placed in service (other than nonresidential real and residential rental property) during the last 3 months of the year have aggregate bases greater than 40% of the aggregate bases of all properties placed in service that year, then a mid-quarter convention will apply instead of the half-year convention. 168(d)(3). Computing the Depreciation Deduction a. 168(a) provides the depreciation deduction for tangible property shall be determined by using: i. the applicable depreciation method ii. the applicable recovery period iii. the applicable convention. b. Begin with the adjusted basis of the property, generally the cost. c. 5-year property s depreciated at the 200% declining balance method. d. Locate table in Revenue Procedure 87-57. e. 168(k) allows for additional first-year depreciation deduction equal to 50% of the adjusted basis of qualified property. Amortization of Intangibles Section 197 a. Allows taxpayers to amortize certain intangibles ratably over a 15-year period. b. Allows for the first time the amortization of goodwill and going concern value. Relationship Between Basis and Depreciation a. Depreciation is the means whereby a taxpayer recovers the cost of property used in a trade or business or investment activity. b. A taxpayer must reduce her basis in a depreciable asset by the depreciation claimed, but not less than the amount allowable. Section 179 Expensing Tangible Personal Property

3.

4.

5.

6.

7. 8.

9.

Basic Income Taxation B | 41

Basic Income Taxation Fall 2011 | Prof. McCormick a. Additional first-year depreciation is applicable to 179 property, which generally is tangible personal property acquired by purchase for use in the active conduct of a trade or business. b. 179 is elective. The Relationship of Debt to Depreciation a. 179 is applicable regardless of whether the taxpayer used her own funds or borrowed funds to purchase the property. It makes no difference whether the taxpayer borrowed the money from a third person or borrowed the money from the party selling the equipment. Revenue Ruling 68-232 a. A valuable and treasured art piece does not have a determinable useful life. Simon v. Commissioner, 1995 a. A deduction for depreciation on tangible property will be available if the property falls within the meaning of recovery property under the Economic Recovery Tax Act of 1981 by being (1) tangible, (2) placed in service after 1980, (3) of a character subject to the allowance for depreciation, and (4) used in the taxpayers trade or business, or held for the production of income. Revenue Procedure 87-56 a. Sets for the class lives of property that are necessary to compute the depreciation allowances available under 168, describing the applicable depreciation methods, applicable recovery periods, and applicable conventions that must be used in computing depreciation allowances under 168. b. Definition of Class Life: i. Means the class life that would be applicable for any property as of Jan. 1, 1986, under 167(m) of the Code. ii. The asset guideline period for the asset guideline class in which such property is classified. ELEMENTS a. Has to be trade/business property or for production of income b. Has to be type of property that wears out c. Have determinative useful life d. Property must not be inventory e. Property has to be susceptible to valuation 168(k) Bonus depreciation a. must buy new property b. Allowed only for new property purchased and placed in service in certain time periods and meeting certain other requirements. To spur economy and purchase of new business equipment. c. 168(k)(5) during certain dates acquire and place in service: i. 9/8/2010 12/31/11 = immediately deduct 100% ii. 1/1/2012-12/31/2012 (k)(1) allows you to deduct 50% of cost iii. After 2012, 168(k) sunsets. d. Factors: i. Recovery period of 20 years or less ii. New property, cant be used iii. Bought and put in service during certain dates iv. 168(k) is automatic. 179 a. election to expense and instead of capitalize (and take depreciation over time). b. Incentive for small business, a method of economic stimulus.

10.

11. 12.

13.

14.

15.

16.

Basic Income Taxation B | 42

Basic Income Taxation Fall 2011 | Prof. McCormick c. Treat the item as a deductible amount up to $500K for the dollar limit in an investment of $2M. i. I.E. Buy $2.1M building, dollar limit is $400K that you can deduct! d. Another Limit: Deduction shall not exceed TI from any T or B of the taxpayer for that year 179(b)(3). Disallowed amount can be carried forward indefinitely. e. Another misc. limit for SUVs. 179(b)(5) limits the cost of any SUV which may be taken into account under 179 to $25K. 17. Process for approaching a depreciation problem: Start with 179, go to 168(k), then 168(a)! Chapter 14 Problems: 1.) With respect to each of the following assets associated with a new luxury hotel, explain whether the asset is depreciable or nondepreciable. (a) The tract of land on which the hotel is constructed. NONDEPRECIABLE land is not depreciable. (b) Fences concrete sidewalks, and landscaping, including trees and shrubbery, on the grounds of the hotel. DEPRECIABLE used in a trade or business and subject to wear and tear and not inventory. 15 year property. (c) Valuable antique furnishings and carpets that are part of the dcor of the public spaces throughout the hotel. DEPRECIABLE. Is it subject to wear/tear/obsolescence? Works of art are not considered depreciable because they are not subject to wear and tear. However, works of art/antique furnishings can be depreciate if it is used as a tool in the trade or business that causes it to be subject to wear and tear. (d) Elegantly framed, but nondescript landscape paintings for each of the guest rooms. These paintings were produced by a business that provides hotels with paintings for decoration purposes. The business hires starving artists to paint the landscapes using colors that will match the dcor of particular hotel rooms. DEPRECIABLE Probably not viewed as true works of art more likely viewed as decorations. The paintings up on the wall in old hotel rooms are faded and dated and therefore are subject to wear. 2.) Liz owns an engineering business. She consults you regarding the deductibility of a sophisticated piece of new equipment she purchased for use in her business on Jan. 19, Year 1. The purchase price for the equipment was $450K. Liz used $50K of her own money and borrowed the other $400K from a local bank to purchase the equipment. The equipment, which is 5-year property under 168, is the only depreciable property she placed in service during the year. Lizs taxable income from her engineering business in Year 1 was $700K, computed without regard to any deductions allowable with respect to the new equipment. (a) Disregarding any application of 179, how much depreciation may Liz claim with respect to the piece of new equipment in Year 1?

Basic Income Taxation B | 43

Basic Income Taxation Fall 2011 | Prof. McCormick Under 168(k) If 2012, she would get 50% of the $225,000. If 2011, she would get 100%! (k) 2011 Deduct 100% all $450K 2012 Deduct 50%, $225,000 Depreciate remaining $225K under 168(a). 2013 deduct 0% under 168(k) Depreciate all $450K under 168(a). Under 168(a) if its 2013, use regular depreciation Regular depreciation under 168(a): (1) method (2) recovery period (3) convention Going to be double declining balance method Recovery period is 5 years so the applicable recovery period is 5 years Convention assume placed in service at same time as purchase, the applicable convention is the half-year convention. 168(d). Go to the 5-year double declining table 1 on page 333 of the text book to get the depreciation rate. $450K x 20% = $90K in Y1 How much depreciation may she claim in Y2? $450K x 32% = $64K in Y2 Y3? $200K x 19.2% = $38.4K in Y3 What will the equipments adjusted basis be at the beginning of Y7 assuming Liz continues to own and use the piece of equipment in her engineering business? Adjusted Basis = Basis Depreciation This is a 5-year property, fully depreciated after the 6th year! Adjusted Basis after Y3 is $200K ($90K - $144K - $43.2K) = $172.8K (b) How much depreciation may Liz claim with respect to the equipment in Y3 if she sells it on Dec. 31 of that year? The taxpayer is allowed a half-year of depreciation in the year of the sale. Thus, $450K x 19.2% = $86.4 x = $43,200 depreciation in Y3 What will her adjusted basis be in the equipment for purposes of computing the gain or loss on the sale of equipment?

Basic Income Taxation B | 44

Basic Income Taxation Fall 2011 | Prof. McCormick AB = Cost Accumulated Dep. $450 90 144 43.2 = $172.8K (c) How would your answer to A change if Liz purchased the equipment in December of Y1? Entitled to deprecation for only the last 1/8 of the year! Mid-quarter depreciation method. Use Table 5, depreciation in year 1 she would get 5%. (d) Assume Liz elects under 179 to deduct the maximum allowable under that provision, what is the total amount which Liz may deduct with respect to the equipment in Y1 under 168 and 179? Under 179 must be tangible property, which 168 applies. Three limitations: (1) A dollar amount depending on the year (2) The deduction is reduced dollar for dollar cost above $500K (3) Is ultimately limited to the taxable income of the business for that taxable year. YEA R 2011 2012 Initial Basis $450 K $450 K $450 K A 179 Deduc t $450K Deduc t $125K Deduc t $0K, Remainin g Basis 0 $325K B 168(k) 0 50% of cost = $162.5 K $0K Remainin g Basis 0 $162.5K C 168(a) 0 20% depreciatio n = 32.5K $90K TOTAL CR in Y1 (A+B+C) $450K ($125+162.5+32.5 ) = $320K Basis = 130K $90K

2013

$450

$450K

3.)

On March 30 of the current year, Pete purchased and placed in service an apartment building for $1.5M. Of that amount, $500K was allocated to the land on which the apartment building was situated and $1M to the building itself. Pete paid $500K down on the property and agreed to pay the balance in installments over the next 20 years. Petes basis in the building is $1M. Land is worth $500K. (a) May Pete claim a 179 deduction with respect to the purchase of the apartment building? Not available because it is not 179(d) property. Not Section 1245 property, and cant be 179 property. Cannot depreciate the building. Depreciable under 167, not under 179. So go to 168(k) & then 168(a).

Basic Income Taxation B | 45

Basic Income Taxation Fall 2011 | Prof. McCormick 168(k): is not available for an apartment building because under 168(d), an apartment building is available to be depreciated for 27.5 years! 168(a): Recovery period is 27.5 years, the applicable depreciation method is the straight line method (under 168(b)(3)(B)). Default is usually double declining balance, but the other method applies. The applicable convention is under 168(d)(2)(a) we use the mid-month convention under 168(d)(4)(b). The mid month convention is used with Table 6 on p. 338 of the text. Bought March 30th of the current year, he would get of march. (b) How much depreciation may Pete claim on the apartment building in the year of purchase? Go to column 3, row 1, depreciation rate is 2.879%. $1M x .02879 = $28.790. How much depreciation may he claim the following year? (Assume that all of the gross rental income Pete receives from the building is rental income from dwelling units.) $(1M 36,360) = Chapter 22: The Interest Deduction - 163 i. What is Interest? a. Points (1% of the amount of the loan) counts as prepaid interest. b. The method of computation does not control its deductibility, so long as the amount in question is an ascertainable sum contracted for the use of borrowed money. c. If loan is treated like dividend, not deductible. d. Personal interest is not deductible!!! e. Investment interest is deductible!! f. Student loan interest is deductible. ii. Deduction of Personal Interest a. The allowance of a deduction for personal interest is an exception to the general rule that personal living expenses are not deductible. b. The general disallowance rule of 163(h)(1) and the definition of personal interest in 163(h)(2). i. Categories of interest excluded: 1. Interest paid or accrued on indebtedness property allocable to a trade or business 2. Any investment interest within the meaning of 163(d) 3. Any qualified residence interest 163(h)(3) 4. Any interest allowable as a deduction under 221. c. Allowed to deduct for qualified residence interest i. It is excepted from the 2% floor ii. It is interest paid or accrued during the tax year on certain acquisition indebtedness and home equity indebtedness secured by the taxpayers principal residence and on one other residence.

Basic Income Taxation B | 46

Basic Income Taxation Fall 2011 | Prof. McCormick iii. Acquisition indebtedness is indebtedness (not in excess of $1M) incurred in acquiring, constructing, or substantially improving any qualified residence of the taxpayer. iv. Home equity indebtedness secured by a qualified residence 1. Limited to the excess of the FMV of the qualified residence over the amount of acquisition indebtedness with respect to such residence. 2. May not exceed $100K 3. The overall limit of indebtedness on a principal and second residence, the interest on which will be deductible, is $1.1M. v. Congress has authorized the deduction of interest on educational loans authorized by 221. 1. Authorizes a limited deduction for interest paid by an individual on any qualified education loan. a. Qualified loans are essentially those incurred to pay higher education expenses tuition, fees, room and board and related expenses of the taxpayer, his or her spouse, or dependents. b. Does not include any indebtedness owed to a person who is related to the taxpayer. 2. Limited to a maximum of $2,500 and is phased out for individuals with modified adjusted gross incomes from $50K to $65K (100K to 130K on joint returns), amounts adjusted for inflation after 2002. iii. Investment Interest 163(d) limits the deduction of investment interest Investment interest for any taxable year cannot be deducted in an amount greater than the taxpayers net investment income. Investment interest is defined as interest paid or accrued on indebtedness properly allocable to property held for investment. Net investment income is the excess of investment income over investment expenses. Investment income includes gross income from property held for investment, i.e. interest, dividends, royalties, annuities not attributable to a trade or business, but generally investment income does not include net capital gain from the disposition of investment property. Qualified dividend income is treated in a manner similar to net capital gain. Investment income does not include income subject to the passive activity rules of 469. Investment expenses are deductible expenses, which are directly connected with the production of investment income. Interest is only deductible to the extent of the taxpayers net investment income. The Jobs and Growth Tax Relief Reconciliation Act of 2003 amended 163(d)(4) to provide that net investment income will include qualified dividend income only to the extent the taxpayer agrees to forego the lower net capital gain rates applicable to these dividends under 1(h)(11). Investment interest expenses which cannot be deducted in one year can be carried-forward. It is not limited by the taxpayers taxable income for the taxable year in which the interest is paid or accrued. iv. Timing Issues and Limitations 461(g)

a. b. c. d. e.

f. g. h. i. j.

k.

a.

Basic Income Taxation B | 47

Basic Income Taxation Fall 2011 | Prof. McCormick i. generally prevents a cash method taxpayer from claiming a current deduction for interest payments which compensate a lender for the use or forbearance of money in future years. Rather, the taxpayer is permitted to deduct only the interest expense related to the current year; the balance may be deducted in the year to which it relates. 1. Places the cash method taxpayer on the accrual method with respect to the deduction of prepaid interest. ii. 461(g)(2) provides an exception for qualifying points paid in connection with the purchase or improvement and secured by the taxpayers principal residence. 1. Huntsman held the taxpayers could not deduct currently points paid to refinance the debt on their personal residence. a. The exception is limited to points paid in respect of financing the actual purchase of a principal residence or financing improvements to such residence. 2. 8th Circuit Reversed: concluding that 461(g)(2) merely requires that indebtedness be incurred in connection in the purchase or improvement of a taxpayers residence. b. 263A i. disallows a current deduction for interest incurred during the production period on indebtedness directly or indirectly attributable to a taxpayers production of certain real or tangible personal property for use in a trade or business or activity conducted for profit. ii. Such interest must be added to the basis of the property and will be recovered through depreciation deductions or when the taxpayer sells or otherwise disposes of the property. c. Payment Issues i. General rule is that the mere giving of a promissory note by a cash method taxpayer does not constitute a payment, even if the note is secured by collateral. ii. A threshold point is generally that when a deductible payment is made with borrowed money, the deduction is not postponed until the year in which the borrowed money is repaid. Such expenses must be deducted in the year they are paid and not when the loans are repaid. d. Davison v. Commissioner, 107 T.C. 35 (1996) i. Interest deductions are not allowed if the funds used to satisfy the interest obligations were borrowed by a cash basis borrower from the same lender to whom the interest was owed. Chapter 22 Problems: 1.) To what extent, if any, may Kevin deduct the following payments which he made during the taxable year? (a) Kevin paid $3K interest on a bank loan used to pay operating expenses in Kevins retail shoe business. Deductible, its a non-personal interest because it is for a trade or business. (b) Kevin paid $500 interest on a loan he obtained to purchase a car for his personal use and $1K in interest on credit cards used to make personal purchases.

Basic Income Taxation B | 48

Basic Income Taxation Fall 2011 | Prof. McCormick Not deductible, personal purposes! (c) Kevin paid $1.2K interest on a bank loan used to pay his daughters college tuition. Can deduct because it is from a bank loan! If his modified AGI is above $50,0000, then the $2500 limit gets reduced. Can deduct until he gets to $65K. What results if Kevin had borrowed the money for his daughters tuition from his (a) brother or (b) uncle? (a) Not deductible. (b) Not considered to be related to him, can count as an education loan! Kevin can deduct $1200 unless his modified AGI is too high. (d) Kevin prepaid the first five years interest (a total of $25K) pursuant to the terms of an installment K Kevin entered into when he purchased land on which he intends to construct a building for his shoe business. Go to 461(g)(1) and capitalize the 5K and deduct over time. He cant make advance payments and deduct interest until their respective year. Exception is points, get to deduct them even though they are prepaid (e) Kevin paid $1K in interest on a loan used to purchase stock. The stock, which currently pays no dividends, was purchased for its growth potential. During the year, Kevin received $300 in interest income and $400 in dividends from other stock he owns. He also paid $50 (fully allowable as an itemized deduction) for a subscription to Investors Weekly magazine. $1K interest on a loan is deductible because it is for investment But, amount is limited under 163(d)(1). Limited to his net investment income for the year, which is here $650! $(300+400)-50 = $650 for net investment income! Must either include the dividend in the limit, which taxes his dividend as ordinary income instead of the capital gains 15%. Otherwise, exclude the $400 in dividends and it would be $300-50 = $250 for net investment income. So, total deductible is $1K - $250 or $650! = taxable amount which would be carried forward. 2.) In January of Y1, Pat bought a home for $600K by paying $100K from his own funds and borrowing the remaining $500K from Home Town Bank. The bank loan was secured by a mortgage on the home. At the time Patrick borrowed the money to purchase the home, it was customary for lenders to charge a borrower points on the amount borrowed. Pat paid the bank $10K points and $50K in interest during Y1. (a) May Pat deduct the Y1 points and the interest? The points are deductible, so long as they do not represent a service charge. The amount must be charged in addition to whatever the normal service charges would be. Threshold question: is this interest or a services charge? If service charge, not deductible. If interest, is this qualified residence interest? The $50K in interest is deductible. This doesnt count as personal interest because it is qualified residence interest! Then analyze the debt. Is it Acquisition Indebtedness or Home Equity Indebtedness? It is Acquisition Indebtedness because it was incurred in acquiring a home! Must be under $1M.

Basic Income Taxation B | 49

Basic Income Taxation Fall 2011 | Prof. McCormick (b) In January of Y8, when Pats home had a value of $800K and the balance on his mortgage was $400K, Pat refinanced the property to take advantage of a 6% per annum interest rate offered by XYZ Mortgage Company. Pat borrowed $600K from XYZ using $400K of the loan proceeds to pay in full the balance owed to Home Town Bank on the original mortgage. Pat used $140K of the loan proceeds to remodel his home and the balance of $60K of the proceeds to pay his car loans and credit card balances. In Y8, Pat paid a total of $39K in interest to XYZ. How much, if any of the interest will be deductible? In order to be deducted, the debt has to be AI or HEI or both. Do the trhee types of debt fit into those categories. $400K = AI, only to extent of the old loan. $140K = AI because it is an improvement and in the definition. Add 400+140 = 540, well within the $1M limit. $ 60 K of personal debt = NOT AI because it did not improve or increase the home. HEI = defined as under $100K. FMB AI = $800K - $540K = $260K. the $60K falls under the $260K of HEI! Deduct the $39K. (c) Assume facts of B. Assume also that in Y8 Pat entered into a 15 year installment contract to purchase a lake cottage which Pat intends to use as his summer home. Pat agreed to pay $500K for the home. Pats obligation to the seller under the K was secured by a mortgage on the cottage. Assume Pat paid $40K down and the first principal payment on the $460K owed to the seller was not due under the contract until Y9. In Y8, however, Pat was required to pay the interest that accrued during that year on the unpaid principal balance. The interest for Y8 amounted to $35K which Pat paid on December 31, Y8. May Pat deduct the $35K interest? $540K AI $60K AI installment contract borrowed $460K The maximum amount youre allowed to have for any period is $1M. He already had $540K of principal residence. Borrowed an additional $460K, adds up to exactly $1M. All $460K meets the definition of AI. All currently deductible. (d) Assume facts of A. In Y5, when the balance owing to HTB was $440K, Pat refinanced the home with HTb solely taking advantage of a significant decrease in home mortgage rates. At the time of the refinancing, Pat owed HTB $10K in interest on the original mortgage. Under the terms of the refinancing arrangement, Pat borrowed $450K from HTB but did not receive any of the loan proceeds. Instead, HTB withheld $440K of the proceeds as payment of the $440K principal balance owing on the original mortgage and withheld the remaining $10K as payment for the $10K in interest which Pat owed the bank on that mortgage. May Pat deduct the $10K in interest that was deemed to be paid as a result of the banks withholding of $10K from the refinancing proceeds?

Basic Income Taxation B | 50

Basic Income Taxation Fall 2011 | Prof. McCormick Received an extension to pay it..safe harbor answer = if the loan is from a new lender, he is viewed as having paid the amount owe and during on the old loan and he gets to deduct it. If the amount borrowed is from the same lender as the initial loan, then the IRS position is that he has not paid the amount and he does not get to deduct it.! Two part test under Davison: (1) look at whether the money he paid was the same money as the money he borrowed under the new loan, if so not good for him if different money new amount. (2) what was the purpose of the second loan. If purpose was to cover 10K for the first loan, hasnt really paid it. If he wins on either prong, he deducts the 10K. IN this situation, he fails, no deduction! Chapter 26: Charitable Deductions (559-582, 585-86) i. Gifts to charities can be tax deductible. ii. Requirements for Charitable Deductions: 170 is a complex maze of rules limiting the deduction. i. Be made to or for the use of a qualified recipient (170(c)) ii. Constitute a transfer of money or property made with no expectation of a return benefit, iii. Actually be paid to the recipient within the taxable year for which the deduction is claimed (170(a)(1)) iv. Not exceed certain percentage limitations (170(b)); and v. Be substantiated by a contemporaneous written acknowledgement prepared by the donee organization and provided to the donor for any contribution of $250 or more (170(f)(8)) or otherwise verified as required by regulations (170(a)(1)). ELEMENTS: i. Must be a contribution ii. Paid in that year iii. To or for the use of a qualified recipient iv. Of what see Substantiation requirements: i. Cash bank record will suffice, need a writing from charity ii. Property need a receipt showing the name, date, and place of location, and description of the property iii. Cash or goods worth more than $250 contemporaneous written acknowledgement of the donee that provides certain information describing the transaction. iv. Property > $5K get a formal appraisal. Amount of deduction i. FMV (minus consideration received back from the charity) ii. 170(e) iii. 170(b) General %age Limitations 1. 170(b)(1)(A) Cash contribution to listed organizations 2. 170(b)(1)(B) Cash contribution to other qualified recipients 3. 170(b)(1)(C) Appreciated property to listed organizations 4. 170(b)(1)(D) Appreciated property to other qualified recipients Bargain Sale to Charity i. Gain from that sale

a.

b.

c.

d.

e.

Basic Income Taxation B | 51

Basic Income Taxation Fall 2011 | Prof. McCormick ii. Determine the charitable contribution deduction iii. Who is a Qualified Recipient? a. Broad range of entities listed in 170(c). b. For an organization to qualify as a 170(c) organization, it must be organized and operated exclusively for religious, charitable, or other specified purposes; its net earnings cannot inure to the benefit of any private shareholder or individual; and its lobbying and political activities must be limited. i. An individual is not a qualified recipient, but a state can be. iv. What is a contribution or gift? a. The Duberstein gift standard has generally been applied in determining whether a charitable contribution has been made. i. A contribution for purposes of 170 is a voluntary transfer of money or property that is made with no expectation of procuring a financial benefit commensurate with the amount of the transfer. 1. Depends upon whether a reasonable person, taking all the facts and circumstances of the case into account, would conclude that enrollment in the school was in no manner contingent upon making the payment, that the payment was not made pursuant to a plan (whether express or implied) to convert nondeductible tuition into charitable contributions, and that receipt of the benefit was not otherwise dependent upon the making of the payment. 2. Presence of one or more rebuts the presumption of a charitable contribution: a. The absence of a significant tuition charge b. Substantial or unusual pressure to contribute applied to parents of children attending a school, c. Contribution appeals made as part of the admission or enrollment process d. The absence of significant potential sources of revenue for operating the school other than contributions by parents of children attending the school e. Other factors suggesting that a contribution policy has been created as a means of avoiding the characterization of payments as tuition. 3. Where a contribution is made to a charitable organization partly in consideration for goods or services, the regulations provide a two-part test. a. The donor must both intend to make, and in fact make, a payment in excess of the FMV of the goods or services. b. The charitable contribution is then limited to the amount of the payment that exceeds that value. 4. Upon the receipt of a quid pro quo contribution in excess of $75, organizations must inform the donor that the deductible contribution is limited to the amount by which the donors contribution to the charity exceeds the value of the goods or services provided to the donor by the charity. a. 6115(b) defines quid pro quo contribution as a payment made partly as a contribution and partly in consideration of goods or services provided by the payor to the donee organization.

Basic Income Taxation B | 52

Basic Income Taxation Fall 2011 | Prof. McCormick i. Exception to religious organizations where the taxpayer receives religious benefits not sold in commercial transactions. ii. Limited to 80% of the contribution. ii. 170(f)(8) generally requires as a condition of deductibility that the taxpayer substantiate, by contemporaneous written acknowledgement from the donee, any contribution of $250 or more. v. Actual Payment Required 170(a)(1) allows a deduction for a contribution the payment of which is made within the taxable year. Imposes a specific timing rule. Any charitable contribution as defined in 170(c) actually paid during the taxable year is allowable as a deduction in computing taxable income irrespective of the method of accounting employed or of the date on which the contribution is pledged. i. Contribution is made upon delivery, or mailing. vi. Limitations on Charitable Deductions Caps the amount which taxpayers may deduct in any one year, 50% of the taxpayers contribution base. i. Contribution base = AGI. Charitable contributions by corporations are limited to 10% of the corporations taxable income. Carried forward for 5 years. As the caps indicate, the amount of the charitable contribution depends: i. On the type of taxpayer ii. The nature of the property contributed, iii. And the nature of the donee organization. vii. Contribution of Services You cannot deduct the value of services contributed to a charity. viii. Contribution of Appreciated Property Generally, if a taxpayer contributes property to a charitable organization, the amount of the contribution will be the FMV of the property. No appreciation is considered in the donation of the property. Under 170(e), in the case of gifts of appreciated property, the charitable deduction will equal the difference between the FMV of the property and the amount of the gain, which would not have been so called long-term capital gain if the property had been sold at its FMV. Essentially the basis amount. i. Applicable in two situations: 1. Selling the property would have generated something other than long-term capital gain, (short-term capital gain or ordinary gain). 2. If item giving is tangible personal property and the donee is not going to keep the property (i.e. sell to make cash). ii. Generally allows a deduction only in the amount of the taxpayers adjusted basis in ordinary income property and taxpayers adjusted basis in property that upon sale would produce short-term capital gain. The charitable deduction must be reduced by the amount of long-term capital gain that would have resulted had there been a sale of the property at the FMV if: i. The property is tangible personal property and the use by the donee is unrelated to the donees tax exempt purpose or function or, as discussed below, the property is disposed of within three years of the contribution; ii. The property is contributed to certain types of private foundations;

a. b.

a. b. c. d.

a. a. b. c.

d.

Basic Income Taxation B | 53

Basic Income Taxation Fall 2011 | Prof. McCormick iii. Or the property is intellectual property or taxidermy property as specified by the statute. Preferential tax treatment is provided to long-term capital gains. i. 1222(3) defines long-term capital gain as a gain from the sale or exchange of a capital asset held for more than 1 year. ii. A capital asset is NOT an item such as inventory, literary and artistic compositions held by a taxpayer who created them, and accounts receivable for services performed. 1. Stocks would be considered a capital asset. ix. Contribution of Partial Interests in Property Generally denies a charitable deduction for a contribution of any interest in property consisting of less than the donors entire interest. i. 170(f)(2) allows for special rules allowing certain transfers in trust to qualify for the charitable deduction. A contribution of the right to use property which the donor owns shall be treated as contribution of less than the taxpayers entire interest in such property. If a taxpayers only interest in property is a partial interest the taxpayer owns only a remainder interest in a tract of land, the contribution of that partial interest to a qualifying charity will entitle the taxpayer to a charitable deduction. i. Cannot purposely divide property to avoid 170(f)(3)(A). Exceptions to the general rule: i. A taxpayer will be entitled to a charitable deduction for: 1. A contribution of a remainder interest in a personal residence or farm; 2. Contributions of undivided portions of the taxpayers entire interest in property; a. Must consist of a fraction or a percentage of each substantial interest or right owned by the donor in the property and must extend over the entire term of the donors interest. 3. And a contribution of a qualified conservation easement. ii. Undivided gifts of a portion of a donors entire interest are not subject to 170(a)(3) which provides a charitable contribution consisting of a future interest in tangible personal property shall be treated as made only when all intervening interest in, and rights to actual possession or enjoyment of, the property have expired or are held by persons other than the taxpayer or related parties. iii. Undivided interest in tangible personal property was drastically changed by 170(o). 1. Immediately before the contribution, all interests in the item are owned by the donor or by the donor and the donee organization. 2. For purposes of determining the deductible amount of each subsequent contribution of an interest in the same item, the FMV of the item for purposes of determining the charitable deduction for the initial fractional contribution; or the FMV of the item at the time of the subsequent contribution. 3. Provides a recapture rule whereby the failure of a donor who has made an initial fractional contribution to contribute all of the donors remaining interest in the item to the same donee before the earlier of 10 years from the initial fractional contribution or

e.

a.

b. c.

d.

Basic Income Taxation B | 54

Basic Income Taxation Fall 2011 | Prof. McCormick the donors death results in the loss of all deductions the donor has taken with respect to contributions of that item. a. Donor would be required to report income in the amount of the deductions claimed. x. Bargain Sale to Charity 1011(b) requires the apportionment of the basis between the charitable contribution and a $25K sale. If property is transferred subject to an indebtedness, the amount of the indebtedness must be treated as an amount realized for purposes of determining whether there is a sale or exchange to which 1011(b) and this regulation apply, even though the transferee does not agree to assume or pay the indebtedness. xi. Substantiation A donor who claims a deduction for a charitable contribution must maintain reliable written records regarding the contribution, regardless of the value or amount of such contribution. i. For a contribution of money, applicable record keeping requirements are satisfied only if the donor maintains as a record of the contribution a bank record or a written communication from the donee showing the name of the donee organization, the date of the contribution, and the amount of the contribution. ii. For a contribution of property other than money, the donor generally must maintain a receipt form the donee organization showing the name of the donee, the date, and the location of the contribution, and a detailed description. iii. A donor of property, other than money, need not obtain a receipt, if circumstances make obtaining a receipt impracticable. 1. Donor must maintain reliable written records regarding the contribution. The required content of such a record varies depending upon factors such as the type and value of property contributed. Property valued at $250 or more: i. No charitable deduction is allowed for any contribution of $250 or more unless the taxpayer substantiates the contribution by a contemporaneous written acknowledgement of the contribution by the donee organization. 1. Must include the amount of cash and a description (but not value) of any property other than cash contributed, whether the donee provided any goods or services in consideration for the contribution and a good faith estimate of the value of any such goods or services. ii. In general, taxpayers are required to obtain a qualified appraisal for donated property with a value of more than $5K and to attach an appraisal summary to the tax return. iii. There is a deduction denied for contributions of clothing or household items unless the clothing or household items are in good used condition or better. 1. Deduction may be allowed for a single item not in good used condition or better if the amount claimed for the item is more than $500 and the taxpayer includes with the taxpayers return a qualified appraisal with respect to the property. xii. Davis v. United States, 1990

a. b.

a.

b.

Basic Income Taxation B | 55

Basic Income Taxation Fall 2011 | Prof. McCormick a. The transfer of funds by parents to their children serving as full-time, unpaid missionaries for a church is not deductible as a charitable contribution to or for the use of the church under 170. xiii. Revenue Procedure 90-12 a. Benefits received in connection with a payment to a charity will be considered to have insubstantial FMV for purposes of advising patrons if the requirements are met: i. The payment occurs in the context of a fund-raising campaign which the charity informs patrons how much of their payment is a deductible contribution, and either ii. The FMV of all of the benefits received in connection with the payment is not more than 2% of the payment, or $50, whichever is less, or 1. The payment is $25 or more and the only benefits received in connection with the payment are token items bearing the organizations name or logo. The cost of all the benefits received by a donor must in aggregate be within the limits established for low cost articles. a. No more than $5 per donor (adjusted for inflation). Chapter 26: Problems 1.) M is very active in his church; he attends weekly service and takes advantage of a variety of programs offered by the church, including religious education programs for his children and family counseling services. He also regularly uses the churchs recreational facilities, which include a well-equipped gym. The church does not charge its members for any of these programs or facilities. As a member, M is expected to contribute at least 5% of his income to the church and to contribute to its overseas mission fund. M does both. Church members receive a monthly magazine published by the church; contributers to the mission fund receive a mission t-shirt. Assuming adequate records to substantiate his donations, may M deduct the amount he contributes? a. He may deduct the amounts he contributes. Is he required to contribute to receive these benefits? No, this takes the case out of the quid pro quo analysis. Church magazine is not significant in terms of value, so it does not count under revenue pro. 90-12 because the value of what he receives is so minimal. (1.170A-1(h)(3) and -13(f) 8 and 9.) There is no requirement to contribute before getting the magazine no quid pro quo. With respect to t-shirt, there is a quid pro quo, but the value of the t-shirt is insubstantial, and it does not count. If there is a quid pro quo going on, then no deduction! Quid pro quo comes from Hernandez. What if, in addition to the above donations, M periodically made small cash contributions to the church ranging from $5 to $20. The only records he had of these cash contributions were entries he made in a financial ledger he maintained. i. NO. Not deductible, must have some sort of written communication authenticating the donation.

Basic Income Taxation B | 56

Basic Income Taxation Fall 2011 | Prof. McCormick

2.) The board of trustees of MLS launches a fund-raising drive to raise money for its Moot Court fund, a fund that will reimburse students and faculty supervisors for travel and related expenses for a moot court competition. (a) M whose son Erik is on the team, donates $1K. Deductible? ii. M is not marking contribution to his son, but she is paying to the general charitable pool. We need to ask 2 questions: (1) is it a contribution? And (2) Is it a qualified recipient? For the qualified recipient analysis the Tripp and Peace cases are applicable: if donor does not earmark who is going to benefit from the contribution, then the donor can deduct. Donor must leave it open for the charity to determine who will get the benefit. In this case, M can deduct. 1. If school is just conduit for her to give money to her son, then it is not a deduction. (b) S, faculty adviser who will accompany the team contributes $1K, deductible i. Faculty adviser: he benefits from his own contribution and he has the power to decide who is going. Under Tripp, if he has the power to earmark, then he will not be allowed to deduct. But if he does not have control over earmarking process, then he can deduct. ii. PROBABLY THOUGH! (c) Local radio station provides free advertising. Advertising is worth $1K. Can the radio station claim a charitable deduction for the $1k? i. Local newspaper provides free advertising worth $1k. The newspaper cant deduct because the value of services provided are not deductible, but the costs incurred from providing services are deductible under 170. and under 1.170A-(1)(g) authorizes a deduction for certain unreimbursed expenses incurred in providing services to charitable organization. ii. Stems from income (advertising produces income), cant deduct! (d) A wealthy supporter handed an envelope containing $200 to each member of the team. Deductible? i. She makes contributions, but it is not a gift to qualified recipient. However, it is possible that she makes gifts for the use of a qualified recipient. We need legal assurance to guarantee that money will be used for the benefit of the school. In Davis case, you can give money to an individual and can still be deductible as long as the person is under legal obligation to benefit a qualified recipient. However, in this case, the children dont have legal obligation to use the money to benefit the school, so this is not deductible.

Basic Income Taxation B | 57

Basic Income Taxation Fall 2011 | Prof. McCormick

3.) Assume P has a contribution base of $250K. What tax consequences to P if she gives $150K to a local college? b. Contribution base = Adjusted Gross Income = $250k. P gives $150k to local college. Limitation on charitable deduction is 50% of contribution base under 170(b)(1)(A). Her limitation is $125k. Therefore, she is entitled to deduct $125k. The remaining $25k is not currently deductible, but she can carry over to the next year subject to the same limitation for 5 years under 170(d)(1)(A). Alternatively, what tax consequences if P gives $10K to a local college and $140K to Private Foundation (a foundation qualified under 170(c)(2), but not within 170(b)(1)(A)(vii)? $10k to local collage and $140k to private foundation qualified under 170(c)(2), but not listed under 170(b)(1)(A)(vii). $10k to local college can deduct the whole thing, 170(b)(1)(A). $140k to Private Foundation 170(b)(1)(A). 170(b)(1)(B) because it does not fit in

The limit of deduction under 170(b)(1)(B) is the less of; i. 30% of contribution base (which is $75k in this case), or ii. excess of 50% of adjusted gross income over the amount of charitable contributions allowable under 170(b)(1)(A) which is $125k - $10k = $115k. She has to take the lesser of these 2 limits, which is $75k. (the other $66k) can be carried forward. 4.) W an attorney makes the following donations to charity during the year. Assuming an adequate contribution base and adequate records to substantiate each donation, what deductions if any may he claim? (a) W owns an office building in an excellent downtown location. A local charity is in need of an office. To assist the charity, W agrees to rent office space to charity for $100 per month, the FRV of the office space is $1500 per month. 170(f)(3)(A): he is not allowed a deduction because he contributes an interest in property, which consists of less than his entire interest. Rule that says if you have a bundle of sticks and give one stick, cant get a charitable deduction.

Basic Income Taxation B | 58

Basic Income Taxation Fall 2011 | Prof. McCormick (b) W serves as a member of an advisory board for the law school where he earned his law degree. The board meets semi-annually. (i) W incurs unreimbursed travel expenses amount to $2K per year as a result of the board meetings. 1. Unreimbursed travel expense related to contribution of services to law school is deductible. 1.170A-(1)(g) The value of service is not deductible under 1.170A-1(g). Therefore, he can deduct $2000 of his travel expenses. (ii) In addition, he periodically handles legal matters for the school and never charges for his time. Services are not deductible (c) W owns a valuable 18th century painting. He executes a document transferring to the local art museum a remainder interest in the painting, which the museum intends to retain as part of its permanent collection. W retains a life estate in the painting. The remainder interest is valued currently at $50K. i. 18th Century painting; he maintains life estate. He cant deduct $50k because he did not give away his entire interest in the painting, hed have to die. Would it make any difference if instead W gave the museum an undivided one-third interest in the painting and executed an instrument giving the museum the right to the use and possession of the painting for the months of June through September of each year months when W is usually away from home on vacation? ii. However, if he gave the museum the right to the use and possession of the painting, 170(f)(3)(B)(ii) allows him to deduct 1/3 the value of the painting. Deduct $16.5K.

Basic Income Taxation B | 59

Basic Income Taxation Fall 2011 | Prof. McCormick

What implications does 170(o) have on Ws gift of a 1/3 interest in the painting? Immediately before the contribution, all interests in the item are owned by the donor or by the donor and the donee organization. 2. For purposes of determining the deductible amount of each subsequent contribution of an interest in the same item, the FMV of the item for purposes of determining the charitable deduction for the initial fractional contribution; or the FMV of the item at the time of the subsequent contribution. 3. Provides a recapture rule whereby the failure of a donor who has made an initial fractional contribution to contribute all of the donors remaining interest in the item to the same donee before the earlier of 10 years from the initial fractional contribution or the donors death results in the loss of all deductions the donor has taken with respect to contributions of that item. a. Donor would be required to report income in the amount of the deductions claimed.

(d) Assume the sale of the painting would have generated a long-term capital gain. What tax consequences to W if in lieu of the transfers in (c) he gives the painting to his church as an item to be sold in the annual church auction? i. Sale of painting given to the church for sale in the auction. We need to look at 3 items: (1) FMV, (2) 170(e)(1)(A) + (B) property that appreciate, and (3) 170(b) (limitation on charitable deduction) If 170(e) applies, the amount of deduction = FMV Gain = AB. 170(e)(1)(A) applies only to short term capital gain and ordinary gain. In this case, 170(e)(1)(A) does not apply because the sale of property generates long term capital gain. 170(e)(1)(B) applies when we have tangible personal property and the charity does not use tangible personal property for the religious or tax exemption purpose. Therefore, 170(e)(1)(B) applies and reduces the deduction only to Adjusted Basis. Reduce FMV Gain (it is long term capital gain!) (170(e)(1)(B) does apply) = answer will always be the basis! 170(e)(1)(B) applies because it is unused for charitable purposes and it is tangible property, deduction is limited to the amount of the adjusted basis in the painting. Would your answer change if the donated property were undeveloped land rather than a painting? What if he gives undeveloped land 170(e)(1)(B) does not apply. Does 170(e)(1)(A) apply? Assuming he owns it for more than a year

Basic Income Taxation B | 60

Basic Income Taxation Fall 2011 | Prof. McCormick ,the sale would have generated long term capital gain and 170(e)(1)(A) would not apply and he get to deduct the full amount. 170(e) does not apply, so he deducts FMV. Then go to 170(b) based on contribution basis, but we are told here that the contribution basis is adequate and there is not reduction in deduction. (e) W gives his alma mater a remainder interest in his personal residence. He retains a life estate in the residence. i. W gives remainder interest in his real property, but retains life estate. 170(f)(3)(B)(i) he can get charitable deduction equal to the value of the reminder interest of the time of giving. (f) W sells $100K worth of XYZ stock to his church for $50K. Ws adjusted basis in the stock is $40K. Sale of $100k worth of stock to his church for $50k; AB = $40k. This is a bargain sale to charity. There are 2 parts to this transaction: (1) Sale taxpayer realizes gain, and (2) Donation charitable donation under 170(a). Sale part: to compute the gain/loss on the bargain sale to charity, we use the formula in 1011(b) to find the adjusted basis for the purpose of this transaction: Special AB / AB Special AB = = AR / FMV (50k / 100k) x 40k = 20k

Therefore, his gain from the sale part of this transaction = $50k - $20k = $30k (Gain Realized = AR Special AB) W will have to report $30k as gain. FMV is $50,000. Now go to 170(e) and then to 170(b). 170(e)(1)(B) does not apply. 170(e)(1)(A) may apply, we dont know when he purchased the stock. If he owned for a year or less, then selling it will result in a short-term capital gain would apply. Basis associated with the donated stock is $20,000, which becomes relevant if 170(e) applies. Gift part: We look at 170(e)(1)(A) + (B) analysis and FMV. Net value he gives away [FMV of what he gives away] = $50k. If he holds it less than a year, 170(e)(1)(A) applies, but not 170(e)(1)(B). If he holds it more than a year, 170(e)(1)(A) does not apply. If 170(e)(1)(A) does not apply, he can deduct $50k. If 170 (e)(1)(A) applies, he can deduct $20k.

Basic Income Taxation B | 61

Basic Income Taxation Fall 2011 | Prof. McCormick

(g) W contributes a parcel of land to the local hospital. The land has a value of $75K and is encumbered by a mortgage of $25K. Ws adjusted basis in the land is $15K. W gives land to hospital. Land values at $75k encumbered by $25k mortgage. His AB = $15k. He is treated as receiving $25k because hospital assumes $25k mortgage. This is also bargain sale to charity; there are 2 consequences: Sale part: AR = $25k Special AB / AB Special AV = = AR / FMV = 5k

(25k / 75k) x 15k

Therefore, Gain = AR Special AB = $25k - $5k = $20k The other $10k of basis will be allocated to the gift part of the transaction. Gift part: 170(a); FMV of what he gives away to the hospital = $50k. If 170(e)(1)(A) or (B) applies, the amount of charitable deduction will be reduced to the remaining basis allocated to the gift part, which is $10k. If 170(e)(1)(A) or (B) does not apply (because he holds the property for more than one year), then he gets to deduct the whole $50k as charitable deduction. If he holds the land less than a year, then he can only get the remaining AB for charitable deduction. If holding more than a year, he get FMV as deduction.

Basic Income Taxation B | 62

Basic Income Taxation Fall 2011 | Prof. McCormick Chapter 31: Capital Gains and Losses Responsible for exam: how capital gains are preferred over ordinary gains answer: special lower tax rates for capital gain of long term capital gain, ordinarty income gets higher tax rate how ordinary losses are preferred over capital losses taxpayer advantage to have an ordinary loss loss offsets a more expensive ordinary income, & the deduction for ordinary losses is generally not limited what extent capital losses can be deducted in the given year? Apply 211(b) to tell how much capital loss can be deducted in the year what happens to any unused capital loss deduction? Deducted to the extent of the capital gains this year, plus up to 3K of ordinary income. Unused loss is carried forward to the next year, indefinitely subject to the same limitation. What the difference between short and long term capital gains and losses? Long-term = held asset for more than a year What are the various tax rates that apply to different kinds or amounts that apply to capital gains? (1)(h) look here! Know that qualified dividends are currently treated as a type of long term capital gain, an adjusted long term capital gain. Compute net capital gain: Total amount of gain which gets preffered rate Determine the character of a gain or loss Ordinary or capital loss was asset sold or exchanged? If not, no capital gain character was asset sold or exchanged a capital asset? Use statutory authority 1221 and judicial authority Capital assets are those that are not listed in 1221. Case: Arkansas Best, Circle K, Cenex, etc. DO NOT NEED TO KNOW: Part that is carried over, do not need to know whether the carried over portion is long-term or short-term. Dont need to allocate capital losses to the capital gains.

1. Preferential Treatment for Long Term Capital Gain a. Gains from the sale of assets like stock constitute income. b. Special tax treatment for the gain recognized on the disposition of certain assets is justified. i. In the Revenue Act of 1921, Congress provided preferential tax treatment for the gains from the sale or exchange of a class of assets Congress characterized as capital assets. 1. A capital asset is property acquired and held by the taxpayer for profit or investment for more than two years (whether or not connected with his trade or business), but does not include stock in trade of the taxpayer or other property of a kind which would

Basic Income Taxation B | 63

Basic Income Taxation Fall 2011 | Prof. McCormick properly be included in the inventory of the taxpayer if on hand at the close of the taxable year. 2. Capital gains rate is 15%. c. Special tax rate is applied to long-term capital gain. Under current law, tax rate ranges from 5% to 28%. Capital loss deduction is limited to the amount of Capital Gains + $3k of ordinary income. Ordinary loss is not limited. Ordinary loss can be used to offset more expensive income. d. The preferential rates of 1(h) only apply when the taxpayer has net capital gain e. equation for NCG: Net Long Term Capital Gain Net Short Term Capital Loss i. net capital gain is defined in 1222(11) as the excess of the net long-term capital gain (the excess of long-term capital gains over long-term capital losses) over the net short-term capital losses (excess of short-term capital loss over short-term capital gain) for the year f. 1(h)(11) for purposes of this subsection, net capital gain means NCG (determined without regard to this paragraph) increased by qualified dividend income i. Technically, preferential treatment exists only when the taxpayer's long term gains exceeds the sum of taxpayers long-term capital losses and net short-term capital losses o Ex1 Short term capital gain only Assume B sold stock for 50K. B's basis in the stock was 40K and she had held the stock for 10 months. B thus had 10k of STCG. Assuming this was her only sale or excahgne during the year, B had no net capital gain ( applying the formula NLTCG NSTCL = NCG, it is obvious there is 0 NLTCG and 0 NSTCL and therefore 0 NCG). Therefore, B would not have been entitled to any preferential treatment on the gain from the stock sale. If B were in a 35% tax bracket, she would have been required to pay 3.5K in federal tax on the stock gain. o Ex 2: Long term capital gain only assume the same facts as Ex 1 except B held the stock for 2 years. Under these circumstances, B would have had a net capital gain of 10k (NLTCG of 10k less NSTCL of 0 = 10k). Under 1(h), even if she were in the 35% bracket, her federal tax on the stock gain would be taxed at a preferential rate in this case under current law at a rate of 15% for a tax of 1500 on stock gain. o Ex 3 Long term capital gain and short term capital gain Assume the facts of Ex 2 except B also had a short-term capital gain of 10k. Her net capital gain would still be 10k (NLTCG of 10k less NSTCL of 0 = 10k) Assuming she were in the 35% bracket, B would pay a tax of 1.5k (.15 x 10k) on the long-term capital gain and would pay a tax of 3.5k on the short-term capital gain. o Ex 4: Long term capital gain and long term capital loss Assume the facts of ex 2 except B also had a long term capital loss of 10K. She would have no net capital gain . 1. Components of Net Capital Gain: o 28% rate gain: Collectibles Gain and Section 1202 gain if anything falls under the 28% rate gain, it will be taxed at a max rate of 28% this category only provides preferential treatment if the taxpayer is in a bracket higher than 28%

Basic Income Taxation B | 64

Basic Income Taxation Fall 2011 | Prof. McCormick 28% rate gain is the sum of collectibles gain and section 1202 gain collectibles gain: 1(h)(5) gain from the sale or exchange of any rug or antique, metal, gem stamp or coin or other collectible as defined by 408(m) which is a capital asset held for more than 1 year section 1202 gain 1(h)(7) in general, it is 50% of the gain from the sale or echange of certain stock described in 1202 o Unrecaptured section 1250 gain 25% rate net capital gain to the extent of so called unrecaptured section 1250 gain is subject to a max rate of 25% in general, unrecaptured section 1250 gain is the long-term capital gain attributable to depreciation allowed with respect to real estate held for more that 1 year o Adjusted net capital gain: 15% or 5/0% rates Net capital gain reduced by a taxpayers 28% rate gain and unrecaptured section 1250 gains equals adjusted net capital gain adjusted net capital gain is subject to a max rate of 15% If the taxpayer is in a 15% or lower tax bracket, they will be taxed as a maximum rate of 5% , a rate that is scheduled to be 0% in 2008 o IMPORTANT Adjusted net capital gain: qualified dividends income Qualified dividend income is treated as part of adjusted net capital gain and thus is taxed at the 15% or 5/0% rate currently applicable to such gains Limitation on the deduction of capital loss o Section 1211(b) specifically provides that the capital losses may be deduction dollar for dollar to the extent of capital gains o for purposes of this limitation, it makes no difference whether the capital gains or losses are long term or short term o Equation: limit for KL = KG +3K of ordinary income o Capital Losses that a taxpayer could not deduct because of the limitation may be carried over to the next tax year 1212(b) Definition of Capital Assets o To be a capital gain, rather than ordinary, (1) assets disposed of have to be capital assets, and (2) assets sold must have been sold or exchanged under 1222. o Treatment of previous income or loss could affect the character of gain or loss o Skim 1231, 1245, and 1250. To determine whether the assets is capital asset: 1221 o Look at 1221 to determine whether it is a capital asset, or o Judicial standard: Arkansas v. Best, Corn Product, and Cenex cases. Corn Products: assets purchased to ensure a supply would be ordinary assets, not capital Cenex: This clarified Corn Products this is the test the assets assuring a source of supply must also be an integral part of the company's inventory purchase system Two requirements o 1) has to be redeemable for the inventory itself o 2) the cost of the inventory has to be directly related to the cost of the assets

165(c)(2) ordinary loss deduction.

Basic Income Taxation B | 65

Basic Income Taxation Fall 2011 | Prof. McCormick 1211(b) limitation on capital loss. Example: 1. OI - $30k KG - $7k KL - $2k 2. OI - $30k KG - $2k KL - $7k The most you can deduct in the current year = $5k and the remaining $2k will be carried forward. 3. OI - $1k KG - $50k KL - $70k The limit on capital loss = $51k because OI in this case is only $1k. Long term v. Short term capital gain Depending on the time the taxpayer holds the assets: if hold > 12 months LTCG. Must hold more than a year. Summary of 1(h) 1(h)(1)(A) 1(h)(1)(B) (ANCG) ANCG + qualified dividend income will be taxed at 5% 1(h)(1)(C) 1(h)(1)(D) 1(h)(1)(E) 15% for any income that falls in 1(h)(1)(B) but this only applies if you are at a tax bracket higher than 15%. 25% rate for unrecaptured 1250 gain 28% tax rate for collectible gain + 1202 gain Total income Net Capital Gain (NCG) = Ordinary income + STCG 5% rate on the amount of net capital gain Limit on capital loss deduction = $7k + $3k of OI = $10k

NCG - 28 gain unrecaptured 1250 gain = Adjusted Net Capital Gain

Most dividends are now taxed at ANCG, not OI. 1(h) ANCG + Div 5% 1(i) 10% and 15%

----------------------------------------------------------------------

Basic Income Taxation B | 66

Basic Income Taxation Fall 2011 | Prof. McCormick ANKG + Div Unrecaptured gain Collectible Example XYZ stock for 6 months STCG Antique Vest for 2 yrs Collectible gain, will be taxed at 28% if you are a high tax bracket. ABC stock held more than 1 yr ANCG, and will be taxed at either 5% or 15% depending on how much OI you have, and thus what tax bracket you are in Painting for 10 months and sell at a gain STCG Formula for Net Capital Gain (NCG) is found in 1222(11). Character of Income To be a capital gain, rather than ordinary, (1) assets disposed of have to be capital assets, and (2) assets sold must have been sold or exchanged under 1222. Goal for exam: 1. Recognize gain/loss 2. Recognize whether it is ordinary or capital a. What type of asset it is 1221 b. Terms that it was held. 3. Capital gain/loss or ordinary gain/loss and at what rate? PROBLEM Definition of Capital Asset T and M, husband and wife, own all the stock of Friendly Car Dealership, Inc., a corp engaged in the business of selling new cars to retail customers. Determine which are capital assets: 1. The new sedan that Friendly just received from the manufacturer and has placed in its sale room. a. 1221(a)(1) used in the inventory No CA. 2. The van that Friendly uses to transport customers to and from its service department. a. 1221(a)(2) Depreciable property used in trade or business No CA. 3. The new station wagon that T and M have just purchased as their family car. a. Car used for personal purpose Yes, because it is not listed under 1221 4. The land and building Friendly owns and uses in the car dealership. a. This is listed under 1221(a)(2) No CA. 5. The vacant land Terry and Margaret purchased as an investment several years ago a. Is a capital asset, personal property not used for business, doesnt fit under (a)(2). b. Managing trades or investment does not rise to the level of a trade or business. 6. The promissory note Friendly received on the sale of a new truck. a. This falls under 1221(a)(4) No CA. 7. T and Ms home. a. T and Ms home: Home is not normally used in the trade or business, so it is not depreciable. So it is CA. 15% 25% 28% 25%, 28%, 33%, and 35%

Basic Income Taxation B | 67

Basic Income Taxation Fall 2011 | Prof. McCormick 8. A painting Ms mother painted and gave to M shortly before the mother died; a second painting by Ms mother, which M received as heir to her mothers estate. a. Painting - 1221(a)(3) deals with artistic composition. When Ms mother gives the painting to M inter vivos gift Ms basis is Ms mother basis under 1015, so this falls under 1221(a)(3)(C), so no CA. b. However, when Ms mother bequeaths the painting in the will to M, this is CA because Ms basis in the painting is FMV, not the basis of the donor, so it is not described in 1221(a)(3). 9. Margarets wedding ring. a. It is a capital asset, because it isnt not described. 10. T and Ms stock in Friendly Car Dealership. a. Stocks in car dealership: because it is not listed under 1221, this is CA. 11. The computer which Friendly formerly owned and used in the dealership, but which T and M now own and use to manage their investments. a. Computer when Friendly owns it No CA because it is under 1221(a)(2) b. Computer when T and M own it managing your own portfolio is not trade or business, so this is not listed under 1221(a)(2), so it is CA. i. Depreciable (1) subject to wear and tear and (2) used in trade or business or used in production of income. Rate Preference for Long Term Capital Gains & Limitation on the Deduciton of Capital Losses During the current year, H and N, husband and wife, had the following transactions: (a) H and N sold their summer home for $240k. They had purchased the home 4 years ago for $170k and never used the home as their principal residence. (b) H sold a painting for $130k. He had purchased the painting as an investment 6 yrs ago for $50k. (c) N sold 100 shares of stock in Z corp for $70k. She had purchased the stock as an investment eight months before for $60k. Explain the tax impact of these transactions on the computations of H and Ns Gross income, assuming that they file a joint return and that they received a total of $300k in compensation from their employment during the tax year. Compute their net capital gain and explain how it will be taxed. 1. Sale of summer home: Gain = $70k and home is CA because it is not listed under 1221. This will be taxed at 15% (1(h)1(c) because they are high-rate taxpayers since they have $300k of Ordinary income. 2. Painting is CA because it is not listed in 1221. Gain = $80k. This is collectible and taxpayers are at high tax bracket, so it will be taxed at 28%. 3. Short term capital gain from stock = $10k will be taxed at an ordinary income rate. Net capital gain 1222(11). Usually a capital asset, owned for less than a year, thus ordinary income!!!! Taxed as ordinary income. NCG = NLTCG - NSTCL NLTCG is defined in 1222(7) as the excess of LTCG over LTCL. NSTCG is defined in 1222(6) as the excess of STCG over STCL

Basic Income Taxation B | 68

Basic Income Taxation Fall 2011 | Prof. McCormick So NSTCG = $0 in this case, and NLTKG = $160k. Therefore, NCG = $160k, but remember to add in qualified dividend income! Remember, 1(h)(11), dividends taxed as net capital gain for purposes of this subsection, net capital gain means net capital gain (determined without regard to this paragraph) increased by qualified dividend income. So, Total NCG in this case is 160K Miscellaneous Capital Gain/Loss Issues S, a newspaper publisher, purchased stock in the ABC Newsprint Co. in order to assure a supply of paper. S later sold the stock at a loss. (a) How would the loss be characterized? a. This is a capital loss. We have to ask if an asset is a CA. We look at 1221 or judicial authorities Corn Product case and Cenex. b. Under the judicial authority, stock in this case is CA because (1) it is not redeemable for news print and (2) the value of her stock is not related to news print. she wants this to be an ordinary loss because that will allow her to deduct the losses from her higher tax bracket ordinary income. She also doesnt want it to be a capital loss because she is limited to capital gains + 3k. This is capital asset Corn Products: assets purchased to ensure a supply would be ordinary assets, not capital, this would be business expenses i. if this was her only authority, she bought this to assure a supply of paper and thus this would be ordinary loss c. Later, Fed circuit decided Cenex; this added additional clarification this is the test, i. the asset assuring a source of supply must also be an integral part of the companies inventory purchase system 1. two requirements a. 1) has to be redeemable for the inventory itself b. 2) the cost of the inventory has to be directly related to the cost of the asset. d. Here, S, under this test, would have to classify this as capital loss

(b) TEST UNDER CENEX: (c) In order for an asset not to be an asset it has to be an integral part of the companys inventory purchase system: Two prongs to meet this: (1) meets if asset is redeemable for the asset itself, (2) cost of inventory must be directly related to the cost of the asset. (d) What if, instead of purchasing stock, S had entered into a 10-yr contract with the newsprint company whereby S agreed to purchase a certain amount of newsprint each year? Assume that S periodically assigns her right to some of the newsprint to other

Basic Income Taxation B | 69

Basic Income Taxation Fall 2011 | Prof. McCormick newspaper publishing companies. How should she characterize the payment received for these assignments? a. Selling contract: in this case, Cenex requirements are satisfied. This is CA because (1) it is redeemable and (2) the cost of contract right is directly connected to the value of news print. If S generates a loss, her loss will be capital loss. Chapter 34: Assignment of Income 2. Who is the proper tax payer to be charged with the income? 3. Two Rules: a. Income is properly taxed to the taxpayer who controls the earning of the income. b. Income from property is properly taxed to the person who owns the underlying property. 4. Statutory Rules; a. 1948 H & W have been permitted to shift income between them to minimize their tax bills, but only if they file jointly. 5. Overview a. In general, the taxpayer who receives income is the taxpayer taxed on it. 6. The Progressive Rate Structure a. The Code has a progressive tax rate structure. 7. Development of Rules Limiting Income-Shifting a. Who? i. Income is taxed to the taxpayer who controls the earning of the income. ii. Income from property is taxed to the owner of the property. 8. Application of the Assignment of Income Rules a. Income is taxable to the person who earns the income, and an anticipatory assignment of personal income will not serve to shift the tax liability for that income. b. Property which is renounced or abandoned cannot be diverted or assigned by the renouncer and cannot be taxed upon the theory that it was received. c. Even where a vow of poverty has been taken, if you earn money in an individual capacity which is given to the religious order you are a part of, since you earned that money, even though you have taken the vow of poverty it is still taxable to you. 9. Income shifting between families and related parties a. Clearly there is an incentive to shift income within families. 10. Lucas v. Earl a. An anticipatory contract cannot prevent a salary from vesting, for tax purposes, in the person who earned it. 11. Helvering v. Horst a. For income tax purposes, the power to dispose of income is the equivalent of ownership of it, so that the exercise of that power to procure payment of the income to another is the equivalent of realization of the income by the one exercising such power.

Basic Income Taxation B | 70

Basic Income Taxation Fall 2011 | Prof. McCormick Chapter 34 Problems: 1.) M owned an interior design business. M earns $250K per year from her business and $40K per year in stock dividends and bond interest. Her intention is to give her son $50K per year while he is in med school. She would like to reduce her tax liability while helping her son.M devised number of plans to shift some of her income and the tax liability to her son. Evaluate the plans: Plan 1: Ms clients typically pay by check or credit card. M is considering asking some to pay by check to make the checks payable to the son rather than her. In this way, she hopes to reduce her tax income while providing financial assistance to her son. Because M is the one who actually earned the income, the income is still taxable to her despite the fact that the actual check was written to the son. No income to R because it is a gift. Will not reduce the amount of income that M is taxed, does not establish the goal. Plan 2: (a) M has thought about employing her son at a salary of $50K per year. Her idea is that the son could work with her during his vacations and could serve as her personal consultant on business matters throughout the year. If Ms son is not doing any work, then the $$50K will still be taxed to M. Additionally, it would depend if his salary is reasonable in light of the type of work he will be performing under 162. If Ms son performs services that are really worth $50K, then the income shifting technique would work. However, it is likely here that the salary is too steep and thus, unreasonable. The excess would be considered a gift and M would not be able to deduct this as a business expense and the income shifting would likely not work. (b) M has contemplated an arrangement where the son would earn the profit associated with sales of furnishings. According to this plan, when a client orders furnishings from her, M would in effect sell those furnishings to her son at her cost and let him complete the sale with the client and thereby earn the profit from the sale. Since the son isnt actually doing any of the work, the income would still be taxed to Martha. Plan 3: (a) M wonders if she could accomplish her purposes by advising each of the companies in which she owns that until further notice, all dividends payable to her with respect to the stock should be paid to her son. Since M is the owner of the property that is generating the income, M will still be taxed. This is a fruit of the trees issue. IN this case, she would need to give the tree or the stock and not the fruits if she wanted to not be the one taxed. Her assigning the dividends does not get rid of her liability as the earner or bearing of the fruits. (b) M is willing to give her son the right to the next five years dividends on her stock; simultaneously, M will give the stock itself to her daughter. Agnes does not have the rights to the dividends, though she has the stock. Thus, taxable to the son. Since the daughter does not have rights to the dividends, and M doesnt have any rights

Basic Income Taxation B | 71

Basic Income Taxation Fall 2011 | Prof. McCormick either, the son is the only party left and thus, he bears the tax liability. Income shifting would not work here. (c) M has thought about selling to her son the right to receive the dividends from her stock for the next five years. She estimates the present value of the next five years dividends from her stock would be approximately $100K. The sales price and interest on the unpaid balance would be payable in ten years. No longer owns the property not taxed to Martha, but to her son because it is his ownership. This is likely that the note will never be paid because M is likely to forgive the loan. If there is no intent to pay, this will be viewed, as a gift and Martha will be taxed. Plan 4: M owns the building in which her office and home furnishings showroom are located. She has almost fully depreciated the building. M would be willing to convey the building to R as a gift and then would lease back the space that she currently uses for her interior design business. M would pay R reasonable rent ($35K) under the lease. Because there is some vacant office space in the building, R could generate an additional $15K in income by leasing to other professionals. Since income from property is taxed to the owner of the property, and Martha conveyed the building to Robert as a gift, Robert will be taxed on the income from the renter because Robert now owns the building. They would likely not be able to shift income here, some courts would ask if there is a valid business purpose in the transfer of the building, if yes, then they can shift the income. Look for valid business purpose, and if she surrendered control over the building. Chapter 37: Tax Consequences of Divorce 1. Section 71 provides for the inclusion of Alimony in income. 2. 215 allows a deduction for alimony paid. 3. Property transfers between spouses and former spouses, incident to divorce, are nontaxable events under the special nonrecognition rule of 1041. 4. Alimony: General Requirements a. 71(a) and 61(a)(3) state that amounts received as alimony or separate maintenance payments are specifically includable in gross income. b. 215 provides an above-the-line deduction for such payments actually paid, essentially mandating the cash method for deduction purposes. c. An alimony payment must meet the following requirements in order to constitute alimony for federal tax purposes: i. The payment must be in cash 1. Payments in the form of property or services do not qualify. ii. The payment must be received by or on behalf of the spouse or former spouse. 1. Cash payments to third parties may in some circumstances qualify. iii. Payment must be made under a divorce or separation instrument. 1. Payments made under an oral agreement, not reduced to writing do not qualify. Ewell v. Commissioner.

Basic Income Taxation B | 72

Basic Income Taxation Fall 2011 | Prof. McCormick 2. Letters must show a meeting of the mind in the separation letter. iv. The divorce or separation instrument must not designate the cash payment as one that is excludable from the gross income of the recipient and nondeductible to the payor. 1. Cant be called something other than alimony. v. If the spouses are legally separated under a decree of divorce or separate maintenance, they must not be members of the same household at the time the payment is made. 1. If not legally separated, payments under a written separation agreement or a support decree may constitute alimony even though the parties are members of the same household. vi. The payor spouse must have no obligation to make payments for any period after the death of the payee spouse. 1. Shifts the tax burden to the payee, not the payor. 2. To determine whether payments satisfy 71(b)(1)(D): a. Courts have looked for an unambiguous condition terminating the payments; that condition may be found either in the plain language of the divorce decree itself or as imposed by operation of state law. b. Where there is no ambiguous termination condition in either the divorce decree or state law, courts have independently evaluated the language of the decree as a whole to determine whether the payments satisfy it. i. If neither part is satisfied, the payments do not constitute alimony. 5. Alimony treatment is favorable under the Tax Code a. Payor gets deduction b. Payee gets increase i. Payor is usually in a higher tax bracket, allows a shifting of income from a high bracket tax payer to a low bracket tax payer. ii. 6. Child Support a. A cash payment must also avoid classification as child support in order to qualify as alimony. 71(c). b. Child support does not get to deducted from the payor, payee does not have to include it! Often an incentive to characterize child support as alimony. c. If both child support is paid and alimony, if the spouse fails to pay the full amount, the allocable amount to each type of support is that the child support full payment for the year is allocated to first, and the rest is put to alimony. d. 71(c)(2)(B) identifies limited specific circumstances under which reductions in payments can be clearly associated with a contingency related to a child, and thus constitute child support rather than alimony. i. Payments otherwise qualifying as alimony are presumed to constitute child support if they are reduced within 6 months of the child turning 18, 21, or the local age of majority.

7. Excess Front-Loading

Basic Income Taxation B | 73

Basic Income Taxation Fall 2011 | Prof. McCormick a. Cash payments which are not fixed as child support and which comply with the rules discussed constitute alimony, includable in the payees income and deductible to the payor. b. The Excess Front-loading rules of 71(f) is an attempt to define alimony in a way that would conform to general notions of what type of payments constitute alimony as distinguished from property settlements and to prevent the deduction of large, one-time lump-sum property settlements. i. Payments characterized as excess alimony payments having been included by the payee and deducted by the payor in a prior year, are recaptured in the subsequent year. 1. The excess payment is deductible by the payee and includable by the payor. ii. Recapture of excess alimony payments can take place in one year only the 3rd post-separation year, a term defined in 71(f)(6). Recapture cannot occur in any other year. Alimony Trusts a. 682 attributes the income from an alimony trust to the wife, the trust beneficiary. No deduction shall be allowed for the husband on account of the trust income taxed to the wife. Dependency Exception a. In the case of a child of divorced or separated parents, the child will ordinarily be a dependent of the custodial parent, as a qualifying child of that parent, or in other words, the parent having custody of the child for the greater part of the year will ordinarily be entitled to the dependency exception for the child. i. The custodial parent is the parent with whom the child resides for the greater number of nights during the calendar year, and the noncustodial parent is the parent who is not the custodial parent.A child resides with a parent for a night if the child sleeps (i) at the residence of that parent (whether or not the parent is there is irrelevant); or (ii) in the company of the parent, when the child does not sleep at a parents residence. ii. If equal for both parents, the parent with the higher adjusted gross income for the calendar year is treated as the custodial parent. Filing Status a. See 2(b). Property Transfers a. US v. Davis: The release of marital rights in return for property resulted in no realized gain or loss to the releasing spouse. i. Holding: A taxpayers transfer of his appreciated property to his former wife, pursuant to their property settlement agreement, in return for her release of her marital rights, produced recognized taxable gain to the transferor-husband. The transferee-wife, in turn, took a FMV basis in the property. ii. REVERSED BY 1041!!!!! b. Under 1041, no gain or loss is recognized on a property transfer between spouses or incident to divorce. i. The transfer instead is treated as a gift, with the transferee taking the transferors basis. ii. Applies not only to transfers incident to divorce but to any transfer of property between spouses, whether in the form of a gift, or a sale or exchange at arms length. iii. A transfer of property on behalf of a spouse to a third party may qualify.

8.

9.

10. 11.

Basic Income Taxation B | 74

Basic Income Taxation Fall 2011 | Prof. McCormick iv. Transferee takes carryover basis! 12. Special Rules Regarding Personal Residence 121 a. 121 allows a taxpayer to exclude up to $250K of gain on the sale of a principal residence. A married couple filing a joint return may exclude up to $500K of gain. b. 121 Special Rules: i. Where property is transferred to an individual in a transaction qualifying under 1041, the period of ownership of that individual for purposes of 121 will include the period the transferor owned the property. ii. Applicable to situations where a taxpayer continues to have an ownership interest in a residence but does not live in the residence because pursuant to a divorce or separation instrument, the taxpayers spouse or former spouse is granted the use of the residence. 1. In these circumstances, the taxpayer will be treated as using the residence while the residence is used by the taxpayers spouse or former spouse. 13. Legal Expenses a. Under the origin-of-the-claim test, legal expenses in connection with a divorce will generally be nondeductible; however, subject to the 2% floor rule of 67, the cost of tax planning advice is generally regarded as deductible, as are the legal expenses attributable to amounts includable in income as alimony. b. 212: pay in connection with the production of income, then the expense is deductible. c. If one spouse pays anothers legal fees, may count as alimony! i. If paid in cash, pursuant to the other criteria, then its alimony! ii. Payor deducts, Payee includes Chapter 37 Problems: F & M were married for 15 years. On Sept. 30, Y1, they separated. M remained in the family home, and F moved into an apartment. They entered into a written separation agreement on Jan. 1, Y2, and a divorce decree, incorporating the terms of that agreement on July 1, Y2. 1.) Assume in Oct. Y1, immediately after the separation, F wrote a letter to M in which he agreed to pay M $2,000 per month starting Oct. 1, Y1. M did not respond to the letter. F made the promised payments. Do the Y1 Payments constitute alimony? No, failed divorce decree! Unacknowledged statement cant be turned into a bilateral agreement as well! 2.) Which of the following payments made pursuant to the Jan 1, Y2 agreement constitute alimony? (a) As of Jan 1, Y2, F has been paying M $2K per month. No. Not designated as alimony. While they are legally separated, they must not live in the same household. (b) On Jan 1, Y2, F transferred to M a parcel of land F purchased years before as an investment ($100K value; $75K basis).

Basic Income Taxation B | 75

Basic Income Taxation Fall 2011 | Prof. McCormick

NO. Not includable as income, its a property transfer. 1041 applies and says property transferred between spouses and ex-spouses incident to divorce does not recognize gain or loss, but takes a carry-over basis (c) As of Jan, Y2, F has been paying Ms monthly rent of $1K. Although not required to do so by the decree, Frank also has been paying Ms utility bills. Rent does qualify as alimony. Treat as alimony, Frank deducts payment M has to include as amounts of income. Utilities: probably not continued after she is dead, assuming that utilities cease when she dies. They are not alimony, not in the agreement!!!! 3.) Assume the Jan 1, Y2 agreement requires F to pay M the following amounts: $5K per month, commencing Jan 1, Y2, and continuing to Jan 1, Y3 at which time payments decrease to $4K per month; on Jan 1 Y4, the payments will decrease against o $1.5K per month, on Jan 1, Y5 payments will decrease to $1K per month and continue through December of Y5 at which time they stop. What are the tax consequences to M and F as a result of the payments? Y2: Y3: Y4: Y5: $5K x 12 months = $60K $4K x 12 months = $48K $1.5K x 12 months = $18K $1K x 12 months = $12K

Issue is excess front-loading of alimony. 71(f)(3): excess payment for post-seperation year (first year that alimony is paid). (4)(A) is $48,000 + (18,000+15000) = Excess payment for second-post seperation year is $15,000.. (3) A + B (the sum of, the average of (I + H) + 15,000) = (i) becomes $25,500 Thus, it is the sum of (i) + (ii) = $25,500 + $15,000 = 40,500 = B. A stands for $60K 60K - 40.5K = $19,500 is the excess!!!! Excess payment for first postseperation year. Overall Excess payment is: the excess payment for Y1 + Y2. Which is $19,500 + $15,000 = $34,500 reported as extra income! M will get a reduction for Y4 of $34,500. F will have to include the $34,500 in his income in Y4. HOW ITS TREATED: Drafting Options: 1. Stated to be child support 2. Stated to be alimony (get favorable tax treatment) a. Reduced when junior = characterized as child support because of the reduction. b. Reduction on March 3, 2015 no mention of child = date may be significant like a child related contingency like a graduation datetax code regulations create two situations under which a presumption will arise that the reduction amount will be child support and not alimony i. Situation 1: one reduction within 6 months before or after a child reaches either 18, 21, or local age of majority. If reduction falls

Basic Income Taxation B | 76

Basic Income Taxation Fall 2011 | Prof. McCormick within the period before and after, then the reduction amount is presumed to be child support and not alimony. One year window. 1. Three ways to rebut presumption: a. Other reason that date was picked b. Local law stating that the alimony has to be reduced on that particular day, then that amount is alimony c. Conclusive rebuttal: if reduction is a complete cessation of alimony, and occurs in the 6th postseparation year, then the reduction amount is not child support, and truly is alimony. ii. Situation 2: occurs if there are two or more reductions occurring within one year before or after two different children or more reach a certain age from 18 to 24 (including 18 & 24) and its the same age for both children. Two year window. 1. First two rebuttals above are available. 2. The conclusive rebuttal is no longer available. 4.) Assume F & M have one child, D , who was 12 on Sept 15, Y1. Their written agreement provides that they will have joint custody of D who will reside part of the year with each parent. The agreement provides: (a) F will pay M $500 per month for child support, commencing Jan 1, Y2, and continuing until D reaches age 18. What are the tax consequences to F & M? Essentially, no alimony here. Child support (b) In addition to the payments in (a), F will pay M $1.5K per month as alimony. What are the tax consequences to F and M as a result of these payments, assuming that pursuant to an agreement, the alimony payments are reduced to $1K per month at the end of Y4 and cease at the end of Y7? Y4 = D will be 15; Y7 = D will be 18. Reached age of majority. Could be characterized as child support, Y4: look at reduction 1. Post-separation year = year that alimony begins to be paid! (c) TO assure payment of the child support and alimony, F will buy and maintain a term life insurance policy on his own life, naming Maureen as beneficiary. F pays $2.5K per year to ABC Life for the policy. What are the tax consequences to F & M? 682 attributes the income from an alimony trust to the wife, the trust beneficiary. No deduction shall be allowed for the husband on account of the trust income taxed to the wife. Deduct as business expense = NO. Deduct as home mortgage interest, charitable = NO. The deduction for alimony = if meets definition of alimony, then he can deduct and M has to keep in income. Counts as payment on behalf of the other spouse and counts as alimony as long as meets other criteria. This is not alimony because it is not made on her behalf here because she does not OWN THE POLICY!

Basic Income Taxation B | 77

Basic Income Taxation Fall 2011 | Prof. McCormick

(d) What additionally facts would you need to determine who is entitled to the dependency exemption for D? Assuming D resides with M for of the year, what is the filing status of F & M? M becomes head of household and claims D, and F is single. How long do they live with each parent. 5.) What are the tax consequences to F & M of the following property transactions? (a) F & M jointly owned some stock (value of $30K, purchased at $45K) and a parcel of land (value of $120K, purchased for $50K, and subject to a $90K mortgage). During Y2 F transferred to M his interest in the land, subject to the mortgage, and M transferred to F her interest in the stock. In Y3, F sold the ABC stock for $30K and M sold the land for $30K in cash, and the purchaser took the land subject to the $90K mortgage. Incident to divorce: 1041 - within agreement and because of cessation of marriage (within 6 years of divorce) This is a transfer between spouses/ex-spouses incident to divorce. Y2: F M for transferring interest in the landher basis in her half interest is $25K; and gets his basis of $25 K.cost basis! M F for $45K.cost basis of the stock. He could recognize a loss upon sale. Y3: F sells stock for $30K.can recognize $15K loss. AR: Land + mortgage of $90K. AB o $50K and a gain of $70K she will have to claim. (b) Instead of transferring to M his interest in the land, F paid M $15K for her interest in the ABC stock and $15K for her interest in the land. F later sold the stock for $30K and the land for $30K in cash and the purchaser took the land subject to the $90K mortgage. F paid M $15K for Stock and $15K for interest in the land. Basis of Stock = $15K; land = $15K + 25K = 40K Stock: sold for $30K - $15K = $15K gain. Land: $30K + $90K Mortgage - $40K = 80K gain. (c) Assume that M and F were the sole shareholders of DEF Corp and that each of them owned 100 shares of the stock. Each of them paid $5K for the stock several years ago. Each block of 100 shares was now worth $50K. Their property settlement agreement provided that F would purchase Ms stock for $50K, but during Y2, F arranged for the purchase to actually be made by DEF as a redemption of Ms stock. F is sole shareholder of DEF. Reg provides that F is to be seen as the person who is liable. What if a tranfer on Franks behalf? transfer between ex psousesM does not recognize gain or loss on stock disposal.

Basic Income Taxation B | 78

Basic Income Taxation Fall 2011 | Prof. McCormick M & F both co-own the company. They have equal ownership in it. Because of the divorce they dont want to work together anymore. F buys M out by paying $50K. Transfer of property between spouses 1041 could apply. DEF redeemed the stock for M and DEF paid her $50K. 1041 states that it must be between ex-spouses, or spouses. DEF is a corporation and no benefit is given because of that. If 1041 does not apply, M must recognize $45k in gain. (d) On Jan 1, Y2, F transferred to M ownership of a car he had purchased for $30K a few years earlier and had been leasing to a local business. The lease term ended Dec 31 Y1. F had claimed $18K depreciation deductions with the car. M uses the car for personal purposes. At the time of transfer, the car was worth $15K and F was owed $1K in lease charges. In addition to transferring to M the car, F assigned to her his right to collect the outstanding lease charges. During Y2, M collected the $1K and later sold the car for $10K. takes basis of it as $0, converts property to the payment, as if sold property, then recognize the gainsimilar to assignment of income principals. 2002-22 says: assignment of income doctrine is generally inappropriate in the context of divorce.w/I one year of divorce and thus incident to divorce. 1041 applies. F has no gain or loss from transferring the car. Sells car for $10K.. has a 2K loss. STOCK TRANFSER: cant treat stock as alimony because its not cash or provided for in the agreement. Must have to do with the cessation of the marriage.transfer is related to the cessation of the marriage if pursuant to the separation instrument and occurs within 6 years of marriage end. 6.) Are the following expenses deductible? (a) M paid her attorney fees of $5K. Depends if they were for tax planning, then yes, usually NOT because it is a personal expense. If fit into 212, then yes. (b) Alternatively, pursuant to the divorce decree, F paid Ms attorney fees of $5K. No. He cannot it is a personal expense, but if it can fit into the deduction for alimony then maybe. Cash, paid behalf of M, pursuant to a decree, stated as alimony, no legal obligation after M diesif satisfied, then yes. (c) F paid his attorney fees of $5K. Arises out of a personal event, cannot deduct. It is a personal expense, but if can fit into 212 (production of income), then yes.

Basic Income Taxation B | 79

S-ar putea să vă placă și